Kettering TMC Modules

¡Supera tus tareas y exámenes ahora con Quizwiz!

The respiratory therapist is called to the emergency department to evaluate a patient who was brought in via an ambulance due to a motor vehicle accident. The patient is cold and clammy with a blood pressure of 82/46 mm Hg. The ECG monitor shows sinus tachycardia with occasional PVC. Which of the following should the therapist evaluate at this time? A. Serum electrolytes B. Cardiac enzymes C. Hb and Hct levels D. 12 lead ECG

Hb and Hct levels

The respiratory therapist is called to the emergency department to evaluate a patient who was brought in via an ambulance due to a motor vehicle accident. The patient is cold and clammy with a blood pressure of 82/46 mm Hg. The ECG monitor shows sinus tachycardia with occasional PVC. Which of the following should the therapist evaluate at this time? A. Serum electrolytes B. Cardiac enzymes C. Hb and Hct levels D. 12 lead ECG

Hb and Hct levels. Look for best test for shock

A 1600 g neonate is on a pressure-limited, time-cycled ventilator at a pressure of 30 cm H2O, IMV rate of 26 /min, FIO2 0.60 and a PEEP of 5 cm H2O. Arterial blood gas results reveal: pH: 7.33 PaO2: 58 torr PaCO2: 45 torr SaO2: 89% Based upon these results, which of the following should the respiratory therapist recommend? A. Increase the PEEP to 8 cm H2O B. Increase the rate to 28/min C. increase the FIO2 to 0.65 D. Increase the pressure to 32 cm H2O

Increase the PEEP to 8 cmH2O

A 3-year-old child with cystic fibrosis is being mechanically ventilated at the following settings: PIP: 34 cm H2O I time: 1.0 second Rate: 22 /min. FIO2: 0.60 PEEP: 6 cm H2O Mode: PC, SIMV Arterial blood gas results show: pH: 7.36 PaCO2: 44 torr PaO2: 49 torr HCO3-: 24 mEq/L The respiratory therapist should adjust the A. PEEP to 8 cm H2O. B. FIO2 to 0.65. C. set rate to 24 /min. D. PIP to 36 cm H2O.

Increase the PEEP to 8 cmH2O

A patient is receiving mechanical ventilation at the following settings: Mode: SIMV Mandatory rate: 12 Total rate: 12 FIO2: 0.50 Set VT: 650 mL Peak flow: 50 L/min PEEP: 5 cm H2O The following pressure-volume loop is observed Which of the following should the therapist recommend? A. Increase the PEEP B. Change to pressure support mode C. Decease the VT D. Increase the peak flow

Increase the PEEP-to help improve compliance

A 6-year-old child who weighs 20 kg (44 lb) is intubated and being mechanically ventilated following an accidental drug overdose at the following settings: Mode: VC, A/C FIO2: 0.80 Set rate: 12 br/min. Total rate: 15 br/min. Tidal volume: 175 mL Arterial blood gas results: pH: 7.48 PaCO2: 31 torr PaO2: 54 torr HCO3-: 22 mEq/L The respiratory therapist should A. initiate 4 cmH2O PEEP. B. decrease the rate to 10 br/min. C. increase the tidal volume to 300 mL. D. increase the FIO2 to 0.85.

Initiate 4 cmH2O

A 6-year-old child who weighs 20 kg (44 lb) is intubated and being mechanically ventilated following an accidental drug overdose at the following settings: Mode: VC, A/C FIO2: 0.80 Set rate: 12 br/min. Total rate: 15 br/min. Tidal volume: 175 mL pH: 7.48 PaCO2: 31 torr PaO2: 54 torr HCO3-: 22 mEq/L The respiratory therapist should A. initiate 4 cmH2O PEEP. B. decrease the rate to 10 br/min. C. increase the tidal volume to 300 mL. D. increase the FIO2 to 0.85.

Initiate 4 cmH2O. patient is overventilating, but ventilating. Move to Oxygenation

A 32-week gestational age infant is receiving mechanical ventilation for hyaline membrane disease. The respiratory therapist suspects that a pneumothorax has developed and performs transillumination, which reveals a brightly illuminated left thorax. The respiratory therapist's FIRST action should be to A. insert a chest tube and connect to a pleural suction system. B. perform a fiberoptic bronchoscopy. C. obtain a STAT chest film. D. suction the infant.

Insert a chest tube and connect to a pleural suction system

During an incentive spirometry treatment using a volume-oriented device, the patient inhales to TLC and exhales slowly to FRC. What other instructions would you give to improve distribution? A. continue with the current instructions B. switch to a flow orientated device C. initiate IPPB D. inspiratory hold

Inspiratory Hold.

The respiratory therapist reviews the ventilator flow sheet for a post-op patient. The ventilator settings have not changed during the past 24 hours. The peak and plateau pressures were recorded as follows: PIP -28, 35, 50 Pplat-23, 30, 44 This information would indicate that A. airway resistance is increasing. B. lung compliance is decreasing. C. the patient needs to be suctioned. D. the patient can begin weaning.

Lung compliance is decreasing

Which of the following is NOT an indication for continuous mechanical ventilation? A. MIP -20 cmH2O B. acute ventilatory failure C. VT < 5 mL/kg D. VC < 10 mL/kg

MIP -20 cmH20

Which of the following formulas will calculate the number of hours an E cylinder will provide oxygen to a patient? A. tank pressure (PSI) x 0.3 / flowrate (LPM) B. flowrate (LPM) x 0.3 / tank pressure (PSI) C. [tank pressure (PSI) x 0.3 / flowrate (LPM)] / 60 D. [flowrate (LPM) x 0.3 / tank pressure (PSI)] / 60

PSI x .3/flowrate/60

The following data has been obtained from a 28-week gestational age infant who was born premature: Color: Cyanotic Chest x-ray: Cardiac enlargement Chest Sounds: Systolic murmur Respiratory rate: 55 Br/min. SpO2: 80% Which of the following diagnostic tests should the respiratory therapist recommend? A. Pre- and post-ductal blood gas studies B. L/S ratio C. New Ballard Score D. Capillary blood gas

Pre and Post ductal blood gas studies.. may have a PDA

An 1800 g neonate in the NICU is being monitored with a TcPO2 electrode. The TcPO2 electrode is reading 42 torr with the temperature set at 38oC. The PO2 from an umbilical artery sample is 72 torr. Which of the following would best explain the difference in these readings? A. There was an error in the arterial blood gas results. B. The TcPO2 electrode needs to be repositioned. C. The TcPO2 electrode temperature setting is too low. D. The TcPO2 electrode has been dislodged.

TcPO2 electrode temp setting is too low

A patient with a history of myasthenia gravis has just been admitted for increased muscle weakness. The respiratory therapist should recommend which of the following diagnostic tests to monitor the patient's drug therapy? A. polysomnography B. electroencephalography C. tensilon challenge test D. methacholine challenge test

Tensilon Challenge Test

While suctioning an adult patient, the respiratory therapist notes that the vacuum regulator is set at 120 mm Hg but only registers 70 mm Hg on the manometer. Which of the following is the LEAST likely cause? A. Leak in the system B. Thumb not completely covering the port C. Loose connection in the collection bottle D. Additional length of suction tubing

additional length of suction tubing

A 26-year-old patient with shortness of breath is admitted to the emergency room. The patient states that he was running in Central Park with a friend and could not catch his breath. Bedside assessment reveals the following data: Pulse: 120 Respirations: 25 br/min Color: pale SpO2: 89% on room air Breath sounds: slightly diminished on the right The respiratory therapist should: A. request a STAT chest x-ray. B. administer 100% oxygen. C. insert a large bore needle into the 2nd intercostal space on the right side in the midclavicular line. D. insert a chest tube into the 2nd intercostal space on the right side in the midclavicular line.

administer 100% O2

Which of the following is the best way to determine the patency of a nasopharyngeal airway? A. instill 5mL of normal saline into the airway B. attempt to pass a suction catheter through the airway C. listen for air flow through the airway D. recommend a sinus x-ray

attempt to pass a suction catheter through the airway

Which of the following is the best way to determine the patency of a nasopharyngeal airway? A. instill 5mL of normal saline into the airway B. attempt to pass a suction catheter through the airway C. listen for air flow through the airway D. recommend a sinus x-ray

attempt to pass a sxn catheter

A patient who recently underwent a total abdominal hysterectomy is complaining of chills and purulent sputum. Breath sounds reveal coarse rales and rhonchi. The results of the CBC indicate a WBC count of 19,000. The most likely diagnosis is that the patient has developed: A. atelectasis B. pneumonia C. hemothorax D. bacterial infection

bacterial infection

A patient involved in a motor vehicle accident has sustained a long bone fracture and remains in traction. The patient suddenly complains of chest pain, and becomes tachypneic and tachycardiac. To determine the cause of the problem the respiratory therapist should recommend A. administering 100% oxygen. B. a V/Q scan. C. streptokinase. D. a STAT chest x-ray.

V/Q scan to check for a pulmonary embolism > patient has been immobile

Which of the following would equal the vital capacity (VC)? A. VT + IRV B. ERV + RV C. IRV + VT + ERV D. IRV + VT + ERV + RV

VC = IRV + VT + ERV

In order to assist a patient with his efforts to quit smoking, the respiratory therapist could recommend? Varenicline Naloxone Bupropion HCI

Varenicline(Chantix) and Bupropion HCI(anti-depressant)

In order to assist a patient with his efforts to quit smoking, the respiratory therapist could recommend Varenicline Naloxone Bupropion HCI A. Yes No Yes B. No No No C. Yes Yes Yes D. No Yes No

Varenicline(chantix)- reduces cravings for tobacco products. Bupropion(Zyban)-anti-depressant reduce withdrawal symptoms

A patient who recently underwent a total abdominal hysterectomy is complaining of chills and producing yellow purulent sputum. Breath sounds reveal scattered crackles. The results of the CBC indicate a WBC count of 3,000 mill/mm3. The most likely diagnosis is that the patient has developed A. atelectasis. B. viral pneumonia. C. influenza. D. pleural effusion.

Viral PNA. High WBC= bacterial, Low WBC =Viral

A patient is receiving continuous mechanical ventilation with 100% oxygen. While suctioning the patient, the ECG shows PVC: The therapist should A. decrease the suction time per pass. B. increase the oxygenation time. C. use a smaller suction catheter. D. decrease the suction pressure.

decrease the suction time per pass

A 53-year-old patient with a history of chronic bronchitis is expected to have difficulty clearing secretions during the post-op recovery days. The respiratory therapist should recommend which of the following to assist the patient with mobilization of their secretions? A. Deep breathing and coughing techniques B. IPPB with albuterol C. Aerosol therapy with racemic epinephrine D. MDI therapy with beclomethasone (Vanceril)

deep breathing and coughing techniques

During cardiopulmonary resuscitation of a 78-year-old patient, the respiratory therapist provides ventilation with a manual resuscitation bag attached to the patient's endotracheal tube. While squeezing the bag, the therapist observes inadequate chest movement. Which of the following is the LEAST likely cause? A. Missing inlet valve B. Defective endotracheal tube cuff C. Incorrect tube placement D. Excessive oxygen flow

excessive oxygen flow.

A respiratory therapist working in the intensive care unit would use a stylet to: A. assist with phonation B. facilitate orotracheal intubation C. maintain a stoma opening D. provide protection to the unaffected lung

facilitate orotrachal intubation

All of the following would be appropriate goals for a pulmonary rehabilitation program for a patient with COPD EXCEPT A. increased exercise tolerance. B. decreased hypoxic symptoms. C. improved resting blood gases. D. improved activities of daily living.

improved resting blood gases- cannot fix the disease from rehab program

An 1800 g neonate is receiving 30% oxygen via an oxyhood with the following arterial blood gas results obtained: pH: 7.34 PaO2: 46 torr PaCO2: 47 torr HCO3-: 22 mEq/L Based upon this information, the respiratory therapist should recommend which of the following at this time? A. Intubate and initiate mechanical ventilation with an FIO2 0.55 B. Increase the FIO2 to 0.35 C. Administer 5 cm H2O endotracheal tube CPAP with an FIO2 0.60 D. increase the FIO2 to 0.50

increase the FiO2 to .50

A 1600 g neonate is on a pressure-limited, time-cycled ventilator at a pressure of 30 cm H2O, IMV rate of 26 /min, FIO2 0.60 and a PEEP of 5 cm H2O. Arterial blood gas results reveal: pH: 7.33 PaO2: 58 torr PaCO2: 45 torr SaO2: 89% Based upon these results, which of the following should the respiratory therapist recommend? A. Increase the PEEP to 8 cm H2O B. Increase the rate to 28/min C. Increase the FIO2 to 0.65 D. Increase the pressure to 32 cm H2O

increase the PEEP to 8 cmH2O

A patient is receiving CPAP therapy and the pressure is fluctuating between +5 and -8 cmH2O. What should the therapist do to stabilize the CPAP therapy? A. increase the flowrate B. decrease the pressure C. check for system leaks D. check for sticking valves

increase the flowrate. This means patient has a larger volume of air being removed compared to replaced

B. Administer 100% oxygen lidocaine can treat PVC but not 1st, atropine for bradycardia, epinephrine-ER drug for pulseless Vtach

While suctioning a patient who is being mechanically ventilated, the respiratory therapist notes the following ECG pattern on the monitor: A) Administer lidocaine B) administer 100% O2 C) administer atropine D) administer epinephrine

A 70 kg (154 lb) patient is receiving mechanical ventilation in the VC, A/C mode following cardiopulmonary arrest. The respiratory therapist notes the following data: Set rate: 10 br/min. Total rate: 10 br/min. FIO2: 0.65 VT: 650 mL pH: 7.30 PCO2: 56 torr PO2: 78 torr HCO3-: 21 mEq/L The therapist should A. institute PEEP therapy at 5 cm H2O. B. increase the set rate to 12 br/min. C. increase the FIO2 to 0.70. D. increase the VT to 800 mL.

increase the set rate to 12

An air/oxygen proportioner is used to provide an FIO2 of 0.55 by a non-rebreather mask. The blender alarm is sounding. Which of the following is the most likely cause? A. Low oxygen inlet pressure B. High air inlet pressure C. Faulty humidifier bottle connection D. Excessive flow to the non-rebreather mask

low O2 inlet pressure.

A patient is on a 28% Venturi mask with the bed covers pulled up over the mask. Arterial blood gas results on the 28% Venturi mask show the PaO2 is 168 torr. The physician asks the respiratory therapist for a recommendation. Which of the following should the therapist recommend? A. Pull the covers down away from the mask and repeat the arterial blood gas in 30 minutes B. Discontinue the oxygen therapy C. Switch the patient to a nasal cannula at 2 L/min D. Report the arterial blood gas results as is

pull the covers down and repeat ABG in 30 min

A mask CPAP system using a threshold resistor valve is not maintaining the desired pressure. To correct this problem, the respiratory therapist should do all of the following EXCEPT: A. check the valve for proper function. B. reduce gas inlet flow. C. reposition the mask and check the seal. D. add a reservoir to the system.

reduce gas inlet flow

A patient with COPD has been admitted for possible pneumonia. The patient is producing moderate amounts of thick yellow sputum and breath sounds are decreased in the right middle lobe. Sputum culture indicates a staphylococcal infection. Which of the following therapies should the respiratory therapist recommend? A. chest physical therapy B. antibiotic therapy C. incentive spirometry D. IPPB

snyibiotic therapy

A patient involved in a motor vehicle accident has sustained a long bone fracture and remains in traction. The patient suddenly complains of chest pain, and becomes tachypneic and tachycardiac. After administering 100% oxygen, the therapist should recommend A. a barium swallow. B. a CT of the chest. C. starting heparin therapy. D. starting lasix.

starting heparin therapy, for coagulation

While performing orotracheal intubation for a patient in respiratory arrest, the respiratory therapist notices that the stylet has advanced from its original position. The respiratory therapist should A. continue with the intubation procedure. B. stop the procedure, manually ventilate the patient, and reposition the stylet. C. retract the stylet immediately. D. remove the stylet and continue with the procedure.

stop the procedure, manually ventilate, and reposition the sty

While performing orotracheal intubation for a patient in respiratory arrest, the respiratory therapist notices that the stylet has advanced from its original position. The respiratory therapist should A. continue with the intubation procedure. B. stop the procedure, manually ventilate the patient, and reposition the stylet. C. retract the stylet immediately. D. remove the stylet and continue with the procedure.

stop, manually ventilate, and reposition stylet

The ICU director wishes to implement a protocol to reduce the risk of ventilator-associated pneumonia (VAP) for patients requiring mechanical ventilation. The respiratory therapist should recommend that the protocol include A. placing the patient in the prone position. B. changing the ventilator circuit each day. C. utilizing a closed-suction catheter system. D. intubating the patient with a Carlens tube.

utilize a closed-suction catheter system

A patient being mechanically ventilated requires endotracheal suctioning. The patient is on high levels of PEEP therapy and has periods of hypotension. The respiratory therapist hyperoxygenates the patient before beginning the procedure. As the therapist disconnects the patient from the ventilator circuit, the above pattern is seen on the ECG monitor. Which of the following could the respiratory therapist recommend to prevent the patient's ECG pattern? A. increase the oxygenation time B. utilize an inline suction catheter C. perform nasotracheal suctioning D. discontinue suctioning the patient

utilize an inline suction catheter

Which of the following items should the respiratory therapist select prior to performing orotracheal intubation? 1. Various sizes of endotracheal tubes 2. Water-soluble lubricant 3. Stylet 4. Magill forceps A. 1 and 2 only B. 2 and 4 only C. 1 and 3 only D. 3 and 4 only

various sizes of ETT and Stylet

A patient who recently underwent a total abdominal hysterectomy is complaining of chills and producing yellow purulent sputum. Breath sounds reveal scattered crackles. The results of the CBC indicate a WBC count of 3,000 mill/mm3. The most likely diagnosis is that the patient has developed A. atelectasis. B. viral pneumonia. C. influenza. D. pleural effusion

viral PNA

70 kg (154 lb) patient is receiving mechanical ventilation in the VC, A/C mode following cardiopulmonary arrest. The respiratory therapist notes the following data: Set rate: 10 br/min. Total rate: 10 br/min. FIO2: 0.65 VT: 650 mL The arterial blood gas results are as follows: pH: 7.30 PCO2: 56 torr PO2: 78 torr HCO3-: 21 mEq/L The therapist should A. institute PEEP therapy at 5 cm H2O. B. increase the set rate to 12 br/min. C. increase the FIO2 to 0.70. D. increase the VT to 800 mL.

increase the set rate to 12

Which of the following values would be used to calibrate a galvanic fuel cell oxygen analyzer? 1. 21% 2. 50% 3. 100% A. 1 only B. 3 only C. 1 & 2 only D. 1 & 3 only

21%-100%

The following data was collected for a patient in the intensive care unit: pH: 7.39 PaCO2: 40 torr PaO2: 90 torr HCO3-: 24 mEq/L BE: +1 mEq/L SaO2: 95% FIO2: .50 Hb: 15g/dl RER: 0.8 PB: 747 torr PECO2: 20 torr What should be reported as the patient's P(A-a)O2? A. 210 torr B. 350 torr C. 500 torr D. 650 torr

210 torr

The following data was collected for a patient in the intensive care unit: pH: 7.41 PaCO2: 40 torr PaO2: 200 torr HCO3-: 25 mEq/L BE: +1 mEq/L SaO2: 98% FIO2: 1.0 Hb: 15g/dl RER: 0.8 PB: 747 torr PECO2: 30 torr What should be reported as the patient's VD/VT ratio? A. 25% B. 40% C. 55% D. 70%

25% PaCO2-PECO2/PaCO2

What tank factor is used to calculate how long an H cylinder will last when the pressure (PSI) and flowrate (LPM) are given? A. .003 B. .03 C. 0.3 D. 3.0

3.0

A patient in the intensive care unit has the following blood gas results: Arterial pH: 7.42 PaCO2: 39 torr PaO2: 90 torr SaO2: 97% HCO3-: 25 mEq/L BE: +1 mEq/L Hb: 14 g/dL Venous pH: 7.39 PvCO2: 46 torr PvO2: 50 torr SvO2: 80 % HCO3-: 25 mEq/L BE: +1 mEq/L What should the respiratory therapist report as the patient's C(a-v)O2? A. 1.8 vol% B. 3.3 vol% C. 5.2 vol% D. 7.3 vol%

3.3% vol SaO2-SvO2 x2, put decimal between two # 97-90=17x2=34=3.4

What total flow is delivered to a patient if the air entrainment mask is set at 35% oxygen and the flowmeter is set at 6 L/min? A. 20 - 25 L/min B. 30 - 35 L/min C.40 - 45 L/min D. 50 - 55 L/min

30-35 L/min

What total flow is delivered to a patient if the air entrainment mask is set at 35% oxygen and the flowmeter is set at 6 L/min? A. 20 - 25 L/min B. 30 - 35 L/min C. 40 - 45 L/min D. 50 - 55 L/min

30-35 L/min.

A patient in the intensive care unit has the following hemodynamic measurements: CVP (mm Hg): 12 PAP (mm Hg): 49/25 PCWP (mm Hg): 18 MAP (mm Hg): 99 Cardiac output (L/min): 7.1 Cardiac index (L/min/m2): 3.7 What is the mean pulmonary artery pressure? A. 14 mm Hg B. 21 mm Hg C. 33 mm Hg D. 40 mm Hg

33 mmHg

A patient in the intensive care unit has the following hemodynamic measurements: CVP (mm Hg): 12 PAP (mm Hg): 49/25 PCWP (mm Hg): 18 MAP (mm Hg): 99 Cardiac output (L/min): 7.1 Cardiac index (L/min/m2): 3.7 What is the mean pulmonary artery pressure? A. 14 mm Hg B. 21 mm Hg C. 33 mm Hg D. 40 mm Hg

33 mmHg

What total flow is delivered to a patient if the nebulizer is set to 40% oxygen and the flowmeter is set at 9 L/min? A. 15 - 20 L/min B. 25 - 30 L/min C. 35 - 40 L/min D. 45 - 50 L/min

35-40 L/min

A patient admitted to the emergency room for an acute exacerbation of asthma requires oxygen therapy. The patient has an irregular respiratory rate and pattern and is dyspneic. Which of the following oxygen administration devices would be the most appropriate? A. 40% air entrainment mask B. nasal cannula at 5 L/min. C. simple oxygen mask at 6 L/min. D. 40 % aerosol mask

40% air entrainment mask- need a high flow for fixed/stable FiO2

A patient will be away from their room for two hours while undergoing a special procedure in the radiology department. They will be using a full E cylinder. What is the maximum flow that the therapist could use without running out of oxygen? A. 2 LPM B. 5 LPM C. 8 LPM D. 10 LPM

5 LPM. PSI x .3/120 min

Which of the following supply pressures would be appropriate for an air/oxygen proportioner? A. 30 psi B. 50 psi C. 100 psi D. 760 psi

50 PSI

Which of the following supply pressures would be appropriate for an air/oxygen proportioner? A. 30 psi B. 50 psi C. 100 psi D. 760 psi

50 PSI. All equipment needs at least 50 to operate

CVP: 16 mm Hg Mean PAP: 30 mm Hg PCWP: 5 mm Hg MAP: 81 mm Hg Cardiac output (QT): 4.0 L/min Cardiac index (QI): 2.1 L/min/m2 What is the pulmonary vascular resistance? A. 200 dynes/sec/cm-5 B. 300 dynes/sec/cm-5 C. 400 dynes/sec/cm-5 D. 500 dynes/sec/cm-5

500 PVR=PAP-PCWP/CO x 80

A patient in the intensive care unit has the following hemodynamic measurements: CVP: 16 mm Hg Mean PAP: 30 mm Hg PCWP: 5 mm Hg MAP: 81 mm Hg Cardiac output (QT): 4.0 L/min Cardiac index (QI): 2.1 L/min/m2 What is the pulmonary vascular resistance? A. 200 dynes/sec/cm-5 B. 300 dynes/sec/cm-5 C. 400 dynes/sec/cm-5 D. 500 dynes/sec/cm-5

500 dynes. PVR= PAP - PCWP/C.O (80)

A 30-year-old male who weighs 68 kg (150 lbs.) has a minute ventilation of 9.0 liters/minute and a respiratory rate of 20 breaths/minute. What is his alveolar minute volume? A. 4 liters/minute B. 6 liters/minute C. 8 liters/minute D. 12 liters/minute

6 lPM. 9/20=450 Vt (-150)=300x20 150xRR=3.0 = 9.0-3.0

A 30-year-old male who weighs 68 kg (150 lbs.) has a minute ventilation of 9.0 liters/minute and a respiratory rate of 20 breaths/minute. What is his alveolar minute volume? A. 4 liters/minute B. 6 liters/minute C. 8 liters/minute D. 12 liters/minute

6 liters/min. Vt=9.0/20=450, 450-150=300x20=6.0.

The following pulmanary function measurements have been determined for a 42-year-old male patient with asthma: VC: 5.1 L. FRC: 2.4 L. ERV: 1.4 L. VT: 0.5 L. IRV: 3.2 L. What should the therapist report as the total lung capacity (TLC)? A. 5.6 L B. 6.1 L C. 7.6 L D. 8.3 L

6.1L IRV + Vt+ ERV + FRC

The following data was collected for a patient in the intensive care unit: pH: 7.38 PaCO2: 40 torr PaO2: 90 torr HCO3-: 25 mEq/L BE: +1 mEq/L SaO2: 98% FIO2: .30 Hb: 15g/dl RER: 0.8 PB: 747 torr PECO2: 10 torr What should be reported as the patient's P(A-a)O2? A. 70 torr B. 160 torr C. 210 torr D. 350 torr

70 Torr

What is the normal range for the mean arterial pressure in an adult? A. 2-6 mm Hg B. 10-20 mm Hg C. 60-80 mm Hg D. 85-100 mm Hg

85-100 mm Hg

What is the normal range for the mean arterial pressure in an adult? A. 2-6 mm Hg B. 10-20 mm Hg C. 60-80 mm Hg D. 85-100 mm Hg

85-100 mmHg

An oxygen conservation cannula would be most appropriate for which of the following patients? A. A patient with pneumonia requiring low flow oxygen. B. A patient with emphysema who is experiencing increased shortness of breath. C. A patient with a severe exacerbation of asthma. D. A patient with pulmonary fibrosis requiring long term oxygen therapy.

A patient with pulmonary fibrosis requiring long term oxygen therapy

Which of the following would be most helpful in determining if a patient is developing ventilatory failure? A. Arterial blood gas B. Blood pressure C. Vital capacity D. Hematocrit

A. arterial blood gas

A 32-week gestational age neonate is receiving CPAP by nasal prongs at 4 cm H2O. The respiratory therapist notices that the pressure manometer is now reading zero. The therapist should A. confirm placement of the nasal prongs. B. recalibrate the pressure manometer. C. remove condensation from the circuit. D. increase the flow to the neonate.

A. confirm placement of the nasal prongs Premature neonate, "reading 0" complete disconnect.

What is the normal PAO2 for a patient breathing room air at sea level? A. 50 - 65 torr B. 90 - 100 torr C. 120 - 140 torr D. 250 - 350 torr

B. 90-100 torr

A patient in the intensive care unit has the following hemodynamic measurements: CVP (mm Hg): 12 PAP (mm Hg): 48/25 PCWP (mm Hg): 18 MAP (mm Hg): 69 Cardiac output (L/min): 3.1 Cardiac index (L/min/m2): 1.7 These results are consistent with: A. Hypervolemia B. Congestive heart failure C. Pulmonary hypertension D. Cor pulmonale

B. CHF. CVP(right heart)-high, PAP(lungs)-high, PCWP(left heart)-high. MAP(after left heart)-low. This is where change happened (LEFT heart)

Which of the following statements are true concerning capillary blood gas sampling versus arterial blood gas analysis? A. Capillary pH will be lower than the arterial pH B. Capillary PO2 will be lower than the arterial PO2 C. Capillary PCO2 will be higher than the arterial PCO2 D. Capillary HCO3 will be higher than the arterial HCO3

B. Capillary PO2 will be lower than Arterial PO2

A newborn infant born at 26 weeks is exhibiting signs of respiratory distress including nasal flaring and retractions. Physical examination reveals heart rate 180/min., respiratory rate 72/min., blood pressure 60/50 mm Hg. The chest radiograph indicates a reticulonodular appearance. The respiratory therapist should recommend: A. nasal CPAP at 10 cmH2O B. assisted ventilation C. 100% oxygen via hood D. Q1H monitoring

Assisted ventilation. Start with low FiO2>2-8 cmH20 CPAP>assisted ventilation

A patient is observed to have an increased respiratory rate and depth of breathing. Their breath has a fruit-like aroma. This would most likely be associated with: A. head trauma B. metabolic acidosis C. drug overdose D. chronic obstructive lung disease

B. metabolic acidosis. patient breathing deeper and faster to help compensate for High CO2

A patient who recently underwent a total abdominal hysterectomy is complaining of chills and purulent sputum. Breath sounds reveal coarse rales and rhonchi. The results of the CBC indicate a WBC count of 19,000. The most likely diagnosis is that the patient has developed: A. atelectasis B. pneumonia C. hemothorax D. bacterial infection

Bacterial infection > increased WBC.. Decreased WBC >Viral infection

A unilateral wheez would most likely indicate which of the following? A) asthma B) atelectasis C) foreign body aspiration D) epiglotitis

C. foreign body aspiration Asthma is bilateral wheeze, atelectasis is diminished, epiglotits is stridor

Pre- and post-bronchodilator spirometry study is performed on a patient with the following results: Pre FVC-69, Pre FEV1-83 Post FVC-70, Post FEV1-81. These results can best be described as A. normal spirometry. B. obstructive only. C. restrictive only. D. both obstructive and restrictive.

C. restrictive only

A patient has been intubated in order to receive volume control ventilation. To inflate the endotracheal tube cuff, the respiratory therapist should add air to the cuff A. until no leak is heard during inspiration. B. and then remove it until a slight leak is heard at peak inspiration. C. to establish a pressure of 20 mmHg. D. to establish a pressure of 40 cmH2O.

C. to establish a pressure of 20 mmHg

A patient is being ventilated with a Servo 300 ventilator in the intensive care unit. The following data is obtained: Mode: SIMV Mandatory rate: 12 b/min Total rate: 12 b/min VE: 8.6 L FIO2: 0.60 PIP: 31 cm H2O PEEP: 10 cm H2O pH: 7.41 PaCO2: 40 torr PaO2: 95 torr SaO2: 96% HCO3-: 23 mEq/L BE: +1 mEq/L A-aDO2: 300 torr C(a-v)O2: 3.6 vol% What should the therapist report as the QS/QT? A. 15% B. 18% C. 20% D. 25%

C. 20% for each 100 of A-a gradient we add 5% and initial 5% for normal shunting

The following data was collected for a patient in the intensive care unit: pH: 7.39 PaCO2: 40 torr PaO2: 90 torr HCO3-: 24 mEq/L BE: +1 mEq/L SaO2: 95% FIO2: .50 Hb: 15g/dl RER: 0.8 PB: 747 torr PECO2: 20 torr What should be reported as the patient's VD/VT ratio? A. 20% B. 35% C. 50% D. 75%

C. 50% PACO-PECO/PACO

Fine crepitant crackles are most commonly associated with which of the following conditions? A. bronchiectasis B. congestive heart failure C. pneumonia D. croup

C. CHF. Fine crackles heard from fluid

A 47-year-old patient admitted for sepsis has a CaO2 value of 12.5 vol%. The patient does not appear cyanotic. Which of the following would be the most important to further evaluate the patient's oxygenation status? A. PaO2 B. SaO2 C. Hb D. PAO2

C. Hb.

While suctioning a patient who is being mechanically ventilated, the respiratory therapist notes the following ECG pattern: A) Normal sinus rhythm B) Left bundle branch block C) Normal sinus rhythm w/multifocal PVC D) Normal sinus rhythm w/artifact

C. Normal sinus rhythm w/multifocal PVC

While assisting the pulmonary physician with rounds in the intensive care unit, the physician comments that he would like to make a change to the patient's ventilator settings. The therapist's most appropriate action should be A. initiate the changes immediately. B. write a verbal order for the change. C. ask the patient. D. ask the physician to write the order.

D. ask the physician to write the order

A patient is diagnosed with a necrotizing pulmonary fistula in the right lung. The physician has requested that the right lung be ventilated at a pressure 10 cmH2O lower than the left lung. The respiratory therapist should recommend providing this type of ventilation via a: A. Transtracheal catheter device B. Double-lumen endobronchial tube C. Cricothyroidotomy D. 14 gauge endotracheal catheter

D. Double-lumen endobranchial tube

A 40-year-old patient who smokes 2 packs of cigarettes per day has a carboxyhemoglobin level of 6.4%. These results are most consistent with A. Severe COPD B. History of dyspnea on exertion C. Need for supplemental oxygen D. Expected level for this patient

D. Expected level for this patient. Normal non smoker 1-2%

A patient has come to the pulmonary function lab for pre-op testing. The patient performs a maximum inspiration followed by a maximum forceful expiration. This procedure would measure: A. RV B. FRC C. TLC D. FVC

D. FVC

A patient is receiving mechanical ventilation at the following settings: Mode: VC, SIMV Mandatory rate: 12 /min Total rate: 12 /min FIO2: 0.50 VT: 700 mL Peak flow: 40 L/min. PEEP: 5 cm H2O The following scalar graphic is observed. Exp Flow doesn't return to baseline before next breath begins Which of the following should the respiratory therapist recommend? A. Increase the PEEP B. Change to pressure control mode C. Decrease the VT D. Increase the peak flow

D. Increase the Peak Flow- patient has AutoPEEP, adjust I:E ratio. Increase flow=decrease Ti=Increase Te

What is the normal range for cardiac index in an adult? A. 0 - 2 L/min/m2 B. 2 - 4 L/min/m2 C. 4 - 6 L/min/m2 D. 6 - 8 L/min/m2

2-4 L/min/m2

Which of the following statements is/are true regarding an Esophageal Tracheal Combitube? 1. Used for pre-hospital airway management. 2. Placed in the trachea with a laryngoscope. 3. It should be replaced with an endotracheal tube as soon as possible. A. 3 only B. 2 and 3 only C. 1 and 3 only D. 1 and 2 only

1 & 3

Which of the following statements are TRUE concerning mechanical deadspace? 1. It will change the VD/VT ratio. 2. It is contraindicated for a patient with a closed head injury. 3. It increases the work of breathing. 4. It should be used in the SIMV mode. A. 1, 2 and 3 only B. 1, 2 and 4 only C. 1, 3 and 4 only D. 2, 3, and 4 only

1, 2, 3

The respiratory therapist notices that a mask CPAP system is unable to maintain the desired level of pressure. Which of the following might be causing the level to not be maintained? 1. Sticking valve in the system 2. Loose fitting mask 3. Faulty humidifier connection 4. Leak around the airway cuff A. 2 only B. 1 & 2 only C. 1, 2 & 3 only D. 1, 2, 3 & 4

1,2,3. there is no cuff with a cpap

In preparing to perform a cardiopulmonary stress test on a 60-year-old man, the respiratory therapist must determine the target heart rate range for the patient. What is the patient's maximum heart rate? A. 130 beats/minute B. 160 beats/minute C. 190 beats/minute D. 220 beats/minute

160 bpm

A patient is breathing a mixture of 70% He / 30% O2 via non-rebreather mask. The oxygen flowmeter indicates a flow of 12 L/min. What is the actual flow of gas to the mask? A. 12 L/min B. 16 L/min C. 19 L/min D. 22 L/min

19 L/min 12 x 1.6(factor for 70/30)

A patient suffering from decompression sickness requires hyperbaric oxygen therapy. The respiratory therapist should initiate therapy at A. 2 ATA. B. 4 ATA. C. 6 ATA. D. 8 ATA.

2 ATA

A patient in the intensive care unit has the following hemodynamic measurements: CVP (mm Hg): 5 PAP (mm Hg): 29/8 PCWP (mm Hg): 8 BP (mm Hg): 130/70 Cardiac output (L/min): 5.1 Cardiac index (L/min/m2): 2.7 What is the mean arterial pressure? A. 15 mm Hg B. 21 mm Hg C. 33 mm Hg D. 90 mm Hg

90 mm Hg

A 62-year-old female has the following arterial blood gas results on room air: pH: 7.37 PaCO2: 38 torr PaO2: 60 torr The co-oximeter is currently down for repair at this time. The physician has asked you to estimate the SaO2. You would estimate the SaO2 to be which of the following? A. 75% B. 80% C. 85% D. 90%

90% 60/90

A patient involved in a motor vehicle accident has sustained a long bone fracture and remains in traction. The patient suddenly complains of chest pain, and becomes tachypneic and tachycardiac. To determine the cause of the problem the respiratory therapist should recommend A. administering 100% oxygen. B. a V/Q scan. C. streptokinase. D. a STAT chest x-ray.

A V/Q scan. long bone fracture and laying around, high risk of Pulmonary Emboli causing deadspace. ventilation w/out perfusion

A. Loss of PEEP bradycardia happened from loss of PEEP, PT hasn't been suctioned yet

A patient being mechanically ventilated requires endotracheal suctioning. The patient is on high levels of PEEP therapy and has periods of hypotension. The respiratory therapist hyperoxygenates the patient before beginning the procedure. As the therapist disconnects the patient from the ventilator circuit, the following pattern is seen on the ECG monitor: A. Loss of PEEP B. Inadequate hyperoxygenation time C. Vagus nerve stimulation D. Normal response to suctioning

A balloon-tipped, flow-directed catheter is positioned in the pulmonary artery with the balloon deflated. Which of the following pressures will be measured by the proximal lumen? A) CVP B) PAP C) PWP D) MAP

A. CVP Swans gans catheter for hemodynamic monitoring. Distal deflated is PAP, Distal inflated is PWP

A 6-year-old child involved in a swimming pool accident is in cardiac arrest. The patient has been intubated with a size 5.5 mm endotracheal tube and has an end-tidal CO2 monitor connected to the airway. During cardiac compression the respiratory therapist notes that the end-tidal CO2 is increasing. This would indicate that A. cardiac output is increasing. B. compression depth is too low. C. endotracheal tube is too large. D. ventilation should be increased

A. Cardiac output is increasing

A patient is receiving IPPB with a Bennett PR-2. The therapist notes that the patient is not reaching the pre-set pressure in an adequate amount of time. The respiratory therapist should A. increase the flow. B. increase the sensitivity. C. adjust the apnea timer. D. decrease the pressure.

A. Increase the flow Most likely cause for not reaching preset pressure is a leak or not enough flow

A multiple trauma victim with internal hemorrhage is being monitored via pulse oximetry. Which of the following conditions would affect the accuracy of her SpO2 readings? A. hypotension B. hyperoxia C. hypocarbia D. hyperthermia

A. Hypotension. Pulse Ox =Perfusion/Shine a light thru the blood. Must have perfusion in order to work, and a light being able to go thru-no dark nail polish

A 44-year-old male patient is being mechanically ventilated with a volume cycled ventilator. The respiratory therapist observes that there is insufficient time for the patient to exhale completely. The therapist should A. increase the flow. B. decrease the minute volume. C. add expiratory retard. D. remove mechanical deadspace.

A. Increase Flow. Decrease I-time, Increase E-time

A 48-year-old postoperative patient in the recovery room is still unconscious. Vital signs are all stable and the SpO2 is 95%. Which of the following should the therapist recommend to prevent a soft tissue obstruction from occurring? A. Oropharyngeal airway B. Nasopharyngeal airway C. Laryngeal mask airway D. Esophageal obturator airway

A. Oropharyngeal airway

A chest X-ray shows increased retro-sternal air, flat hemidiaphram, decreased movement, and no vascular markings on the right side. These signs would be most likely associated with: A. pneumothorax B. pleural effusion C. pneumonia D. flail chest

A. PNX. Increased air between lungs and chest

The FRC (functional residual capacity) is measured on a patient using the helium dilution technique and the result is 3.0 liters. The same patient is then tested in a body box and the result is 4.5 liters. Which of the following would account for the difference? A. The patient has significant non-ventilated lung volume B. There was a leak during the helium dilution procedure C. The patient did not perform the maneuver properly D. Hysteresis is present

A. Patient has significant non-ventilated lung volume (RV) Helium dilution doesnt measure RV

The results of a patient's chest radiograph reveal the presence of a left basilar free fluid accumulation with a meniscus formation. Physical examination of the chest indicates a dull percussion note on the left and trachea shifted to the right. These results are consistent with which of the following conditions? A. Pleural effusion on the left B. Basilar pneumonia of the left lung C. Atelectasis of the left lung D. Pneumothorax in the left lung

A. Pleural effusion on the left. Menisucs means fluid outside of lung in pleural space. Dull is fluid filled organ. Trachea shifts away=problem outside the lung

A 1000 g neonate is stable in the NICU. Which of the following should the respiratory therapist use to monitor the neonate's overall cardiopulmonary status? A) TcPCO2 and TcPO2 monitor B) Arterial blood gas analysis Q4 hours C) SpO2 monitor D) Capillary gas analysis Q8 hours

A. TcPCO2 and TcPO2 monitor is noninvasive and continuously monitoring ventilation/O2 ABG is invasive and Q4, SpO2 monitor only measures O2, Cap Gases not accurate(worst answer)

Which of the following describes the proper technique when using a stylet? A. The distal end should be recessed at least 1 cm from the tip of the endotracheal tube B. The distal end should be positioned at the level of the beveled end C. The distal end should be proximal to the "Murphy's Eye" D. The distal end should be positioned proximal to the cuff

A. The distal end should be recessed at least 1 cm from the tip of the endotracheal tube

which of the following describes the proper technique when using a stylet? A. The distal end should be recessed at least 1 cm from the tip of the endotracheal tube B. The distal end should be positioned at the level of the beveled end C. The distal end should be proximal to the "Murphy's Eye" D. The distal end should be positioned proximal to the cuff

A. The distal end should be recessed at least 1 cm from the tip of the endotracheal tube

The respiratory therapy supervisor is paged STAT to the operating room to respond to an alarm. The operating room nurse informs the therapist that the nitrogen zone valve alarm is sounding. The nitrogen system to the OR is supplied by a manifold system. Which of the following should the respiratory therapist look for FIRST? A. a leak B. an obstruction in piping system C. a back up bank of cylinders D. the alarm switch

A. a leak

A patient with known reversible airway disease takes two puffs of albuterol from his metered-dose inhaler. The respiratory therapist measures the patient's peak flow following the administration of the medication and determines that the peak flow has increased only minimally. The therapist should A. add a spacer to the metered-dose inhaler. B. change the medication to levalbuterol C. administer the medication by small volume nebulizer. D. contact the physician concerning the care of this patient.

A. add a spacer to the metered-dose inhaler

A 48-year-old female patient is intubated and being mechanically ventilated with a volume-cycled ventilator. The following parameters were obtained from the ventilator flow sheet: Time SpO2 Peak Pressure Plateau Pressure PEEP 2:00 pm 93% 30 25 5 3:00 pm 88% 42 30 10 Based on this information, the respiratory therapist should conclude that A. airway resistance is increasing. B. compliance is decreasing. C. SpO2 measurement is inconsistent with the data obtained. D. a decrease in shunt is occurring.

A. airway resistance is increasing. PIP= Raw/Cst, Pplat=Cst. Raw increased from 5-12

A 46-year-old female patient weighing 65 kg (143 lb) is on mechanical ventilation following a motor vehicle accident. Ventilator settings are as follows: FIO2: 0.40 Mode: VC, SIMV Set rate: 10 br/min. Total rate: 16 br/min. VT: 500 mL Pressure support: 25 cm H2O pH: 7.51 PCO2: 29 torr PO2: 140 torr HCO3-: 22 mEq/L Based upon this information, what should the respiratory therapist recommend at this time? A. Decrease pressure support to 20 cm H2O B. Decrease FIO2 to 0.30. C. Place on a T-piece at 50% oxygen. D. Decrease set rate to 8 br/min.

A. decrease pressure support to 20 cmH2O. Set high at 25 cmH2O. Patient is being overventilated and over Oxygenated. So decrease what is set high on ventilator

A 60-year-old male patient weighs 80 kg (176 lb) and is on a volume-cycled ventilator at a set VT of 800 mL, a peak pressure reading of 65 cm H2O, exhaled VT measured at 760 mL, peak inspiratory flowrate is 60 L/min., a plateau pressure reading of 58 cm H2O and a PEEP of 5 cm H2O. Which of the following is most likely occurring at this time? A. Dynamic compliance is decreasing B. Static compliance is increasing C. Airway resistance is increasing D. Thoracic compliance is increasing

A. dynamic Cst is decreasing. PIP and Pplat high, increasing pressures= decreasing Cst. Static Cst increase = decrease Pplat. If PIP - Pplat is >10 Raw is high

All of the following could cause a capnography reading to change from 36 torr to 30 torr EXCEPT A) endotracheal tube positioned in the right mainstem bronchus B) hyperventilation C) pulmonary emboli D) hypovolemia

A. endotracheal tube positioned in the right mainstem bronchus. This will cause PeTCO to increase

A 80 kg (176 lb) patient with pulmonary fibrosis is being mechanically ventilated with a volume-controlled ventilator at the following settings: Mode: SIMV VT: 700 mL Mandatory rate: 15 Total rate: 15 FIO2: 0.45 PEEP: +10 I:E Ratio: 1:2 PIP: 50 cmH2O Arterial Blood Gas analysis reveals the following: pH: 7.34 PaCO2: 42 torr PaO2: 55 torr HCO3-: 22 mEq/L SaO2: 88% The respiratory therapist should recommend: A. increasing the FIO2 to 60%. B. decreasing the respiratory rate. C. increase the PEEP level to 20 cm H2O. D. changing to pressure control mode.

A. increase FiO2 to 60%

A patient with CO poisoning is receiving oxygen therapy by a non-rebreather mask at 10 L/min. The respiratory therapist notices that the reservoir bag on the mask is emptying completely on inspiration. The therapist should A. increase the flow. B. switch to a partial rebreather mask. C. initiate mask CPAP therapy. D. switch to a venturi-mask.

A. increase the flow

A patient has been admitted to the emergency department after being rescued from a house fire. The patient has second-degree burns on his neck and chest. The respiratory therapist should administer the oxygen therapy via A. non-rebreather mask. B. aerosol mask. C. face tent. D. transtracheal oxygen catheter.

A. non-rebreather mask

A 26-week gestational age infant requires intensive monitoring and care in the NICU. Which device would be most appropriate for maintaining a neutral thermal environment for this infant? A. radiant warmer B. isolette C. croupette D. bassinet

A. radiant warmer neonate needs intensive monitoring requires easier access

A patient in the intensive care unit has the following data obtained: pH: 7.41 PaCO2: 40 torr PaO2: 235 torr SaO2: 99% HCO3-: 23 mEq/L BE: +1 mEq/L FIO2: 1.0 VD/VT: 0.35 Hb: 15 g/dL RER: 0.8 PB: 747 What should the respiratory therapist report as the P(A-a)O2? A. 210 torr B. 415 torr C. 535 torr D. 620 torr

B. 415 torr 7xFiO2 - (CO2 +10)=PAO2

A newborn infant born at 26 weeks is exhibiting signs of respiratory distress including nasal flaring and retractions. Physical examination reveals heart rate 180/min., respiratory rate 72/min., blood pressure 60/50 mm Hg. The chest radiograph indicates a reticulonodular appearance. The respiratory therapist should recommend: A. nasal CPAP at 10 cmH2O B. assisted ventilation C. 100% oxygen via hood D. Q1H monitoring

Assisted ventilation

While suctioning an adult patient, the respiratory therapist notes that the vacuum regulator is set at 120 mm Hg but only registers 70 mm Hg on the manometer. Which of the following is the LEAST likely cause? A. Leak in the system B. Thumb not completely covering the port C. Loose connection in the collection bottle D. Additional length of suction tubing

Additional length of suction tubing

A 2-year-old child has been admitted to the pediatric unit with a barking cough and mild stridor at rest. The pulse oximeter displays an SpO2 of 87% on room air. The physician asks the respiratory therapist to recommend the most appropriate oxygen delivery device for this child. The therapist should recommend a/an A. aerosol mask. B. nasal cannula. C. oxygen hood. D. venturi-mask.

Aerosol Mask > cool aerosol(decrease swelling) + O2(increase Sats)

A 75-year-old patient with COPD is receiving oxygen at home by nasal cannula with a bubble humidifier. How should the respiratory therapist instruct the patient to clean his humidifier? A. Place it on the top shelf of the dishwasher. B. Soak it in an acetic acid solution for 20 minutes and rinse with water. C. Rinse it with distilled water and allow to air dry. D. Soak it in an alkaline glutaraldehyde solution for 30 minutes and rinse with water.

Acetic Acid Solution (White vinegar:Water

While performing routine ventilator parameter checks on a patient weighing 80 kg (176 lb), the respiratory therapist notes the following: Mode: VC, SIMV Set rate: 12 /min. Total rate: 12 /min. FIO2: 0.65 VT: 700 mL The arterial blood gas results are as follows: pH: 7.47 PaCO2: 34 torr PaO2: 55 torr HCO3-: 23 mEq/L Which of the following should the therapist recommend at this time? A. Decrease rate to 10 /min. B. Add 6 cm H2O PEEP C. Increase FIO2 to 0.70 D. Decrease VT to 600 mL

Add 6 cmH2O

While performing routine ventilator parameter checks on a patient weighing 80 kg (176 lb), the respiratory therapist notes the following: Mode: VC, SIMV Set rate: 12 /min. Total rate: 12 /min. FIO2: 0.65 VT: 700 mL The arterial blood gas results are as follows: pH: 7.47 PaCO2: 34 torr PaO2: 55 torr HCO3-: 23 mEq/L Which of the following should the therapist recommend at this time? A. Decrease rate to 10 /min. B. Add 6 cm H2O PEEP C. Increase FIO2 to 0.70 D. Decrease VT to 600 mL

Add 6 cmH2O PEEP

All of the following could be causing the peak pressure alarm to sound on a volume-cycled ventilator EXCEPT: A. airway resistance has decreased B. the patient needs to be suctioned C. compliance has decreased D. an insufficent amount of time for exhalation

Airway resistance decreased will not sound high pressure alarm

Which of the following are true statements concerning an automated medication dispensing system? 1. Maintains accurate information about when medication was dispensed. 2. Allows multiple practitioners to access patient medication. 3. Assists with management of medication inventory. A. 2 only B. 1 and 3 only C. 1 and 2 only D. 1, 2, and 3

All of the above

Which of the following are true statements concerning an automated medication dispensing system? 1. Maintains accurate information about when medication was dispensed. 2. Allows multiple practitioners to access patient medication. 3. Assists with management of medication inventory. A. 2 only B. 1 and 3 only C. 1 and 2 only D. 1, 2, and 3

All of the above

The advantages of an electronic medical record include all of the following EXCEPT A. allows the respiratory therapist to treat multiple patients simultaneously. B. improves the accuracy of charting the results of therapy. C. data entry is more efficient. D. prevents unauthorized access to medical information.

Allows the respiratory therapist to treat multiple patients simultaneously

A bilateral wheeze would most likely indicate which of the following? A. asthma B. Atelectasis C. Foreign body aspiration D. Epiglottitis

Asthma

A bilateral wheeze would most likely indicate which of the following? A. Asthma B. Atelectasis C. Foreign body aspiration D. Epiglottitis

Asthma

A 64-year-old patient is being resuscitated for full cardiopulmonary arrest. After several unsuccessful attempts, the patient is orally intubated with a size 7.0 mm endotracheal tube. The physician is unable to establish a peripheral or central intravenous line. The ECG monitor now shows bradycardia: The respiratory therapist should recommend administration of A. lidocaine by small volume nebulizer. B. atropine through the endotracheal tube. C. epinephrine by intra-cardiac injection. D. amiodarone by intraosseous injection.

Atropine through ET Tube. Epinephrine for Emergency(jump start heart). Amiodarone-PVC

A 64-year-old patient is being resuscitated for full cardiopulmonary arrest. After several unsuccessful attempts, the patient is orally intubated with a size 7.0 mm endotracheal tube. The physician is unable to establish a peripheral or central intravenous line. The ECG monitor now shows the following rhythm: Bradycardia: The respiratory therapist should recommend administration of A. lidocaine by small volume nebulizer. B. atropine through the endotracheal tube. C. epinephrine by intra-cardiac injection. D. amiodarone by intraosseous injection.

Atropine through ETT

An H cylinder of oxygen has 1200 psi remaining in the tank. How long will it take to decrease to 200 psi if the flow is 5 LPM? A. 1 hour B. 10 hours C. 60 hours D. 600 hours

B. 10 hours PSI x 3.14/5LPM=600 hours/60 min

What is the normal range for cardiac index in an adult? A. 0 - 2 L/min/m2 B. 2 - 4 L/min/m2 C. 4 - 6 L/min/m2 D. 6 - 8 L/min/m2

B. 2-4 L/min/m2

A 68 kg (150 lb) patient has a spontaneous tidal volume of 450 mL and is breathing at a rate of 12 breaths/min. What is their minute alveolar ventilation? A. 1.8 L/min B. 3.6 L/min C. 5.4 L/min D. 8.6 L/min

B. 3.6 LPM Alveolar Ventilation= Vt-DeadSpace x R

A patient is found in full cardiopulmonary arrest. CPR is started and the patient is orally intubated with an EtCO2 monitor attached. Which of the following EtCO2 patterns would the respiratory therapist expect to observe on the monitor? A. Initially high, then falling B. Initially low, then rising C. Initially high, stays high D. Initially low, stays low

B. Initially low, then rising. because lungs are empty then during CPR perfusion increases PCO2

A chronic hypercapnic patient enters the emergency room complaining of shortness of breath. The patient is coughing up inspissated, pale, yellow secretions. Which of the following would you recommend at this time? A. Sputum culture and sensitivity B. Oxygen at 2 LPM via nasal cannula C. A-P and lateral chest x-ray D. Arterial blood gases

B. Oxygen at 2 LPM via nasal cannula, do this first out of all options

While assessing a patient's breath sounds the respiratory therapist notes that when the patient is instructed to say the letter "E", it comes through the stethescope sounding like "aaaahhh". This change in the sound is associated with which of the following conditions? A. Pleuritic inflammation B. Pneumonia C. Bronchospasm D. Epiglotitis

B. Pneaumonia. Pleuritic inflammation-pleural friction rub, bronchospasm-wheezing, epiglotitis-stridor

What is the function of the one-way valve on a mouth-to-valve mask resuscitation device? A. Increase the delivered FIO2 B. Prevent the patient from exhaling back C. Prevent the patient from breathing room air D. Control the flow of gas to the patient

B. Prevent the patient from exhaling back

The respiratory therapist is assisting the physician with the insertion of a pulmonary artery catheter. The therapist and physician have proper sterile gloves and gown, and a sterile field is arranged. As the therapist hands the pulmonary artery catheter to the physician, the therapist drops the catheter onto the sterile field. The respiratory therapist should A. run to get another sterile catheter. B. proceed with the insertion of the catheter. C. rinse the catheter with sterile water. D. visually inspect the catheter for any debris.

B. Proceed with the insertion of the catheter. Everything is still sterile

The results of a patient's spirometry are reported as follows: FVC: 3.00, 1.90 FEV1: 2.09, 1.85 Which of the follow is the most appropriate interpretation? A. Normal B. Pulmonary Fibrosis C. Emphysema D. Chronic Bronchitis

B. Pulmonary Fibrosis

A patient is admitted to the ICU complaining of nausea and chest pain. A nasogastric tube has been inserted to help relieve the nausea. The patient was started on lasix and nitroglycerin. Which of the following should the respiratory therapist monitor closely to identify side effects at this time? A. Cardiac enzymes B. Serum electrolytes C. Arterial blood gases D. Digitalis levels

B. Serum Electrolytes-fluid loss (lasix). Cardiac enzymes expensive used to confirm MI. Digitalis-nothing in question about this

Prior to performing spirometry on a pre-op patient, the respiratory therapist calibrates the spirometer using a 3.0 L super syringe. The three volumes achieved are: 2.80 L, 2.80 L, 2.79 L. Based upon the information obtained the therapist should conclude that the 1. spirometer is precise. 2. spirometer is accurate. 3. super syringe was advanced too slowly. 4. spirometer may have a leak. A. 1 and 3 only B. 1 and 4 only C. 2 and 3 only D. 2 and 4 only

B. Spirometer is precise and may have a leak, precise means consistent

The blood pressure obtained from the arterial line is higher than the blood pressure obtained from a sphygmomanometer. Based upon this information, the respiratory therapist should conclude that A. non-compliant tubing is being used. B. the transducer is placed too low. C. the patient was lying flat during the measurement of the arterial line pressure. D. the transducer dome contained air bubbles.

B. The transducer is placed too low- placed below heart, is downhill flow = more pressure

While reviewing a patient's medical record the respiratory therapist notes that the patient has a Hb level of 6 g and SpO2 of 94%. The patient is receiving oxygen by simple mask & is complaining of shortness of breath. The respiratory therapist should recommend A. Changing to a non-rebreather mask B. Administering whole blood C. Initiating PEEP therapy D. Discontinuing oxygen therapy

B. administer whole blood

A spontaneously breathing 76-year-old patient who weighs 60 kg (132 lb) is on an FIO2 of 0.45 via a trach collar. He has had a change in his tidal volume from 600 mL to 300 mL and his respiratory rate went from 12 /min to 24 /min. Which of the following changed due to the change in the tidal volume and rate? A. Minute ventilation B. Alveolar ventilation C. Anatomical deadspace D. Mechanical deadspace

B. alveolar ventilation, pre-600x12=7.2, post-300x24=7.2. MV stayed same means Alveolar changed

A 3-year-old child is admitted to the emergency room with a sudden, persistent, dry cough. Chest x-ray results reveal a normal inspiration with only one lung expanded on expiration. Which of the following would you recommend at this time? A. Endobronchial intubation B. Bronchoscopy C. Arterial blood gas analysis D. Lateral neck x-ray

B. bronchoscopy. Lower airway trapping-may be foreign body obstruction

A nasopharyngeal airway is indicated for which of the following patients? A. unconscious patient with a closed head injury B. conscious patient with an ineffective cough C. alert patient who is expectorating a large amount of secretions D. uncooperative patient

B. conscious patient with an ineffective cough

All of the following would be associated with the presence of a pnewumothorax EXCEPT: A) tracheal deviation B) dull percussion C) absent breath sounds D) respiratory distress

B. dull percussion- makes a high pitched tympanic sound tracheal deviation pushed away from PNX

A 16-year-old male patient involved in a motorcycle accident presents to the ER with massive maxillary and nasal trauma. Which of the following devices would be most appropriate for maintaining the patient's airway? A. nasal endotracheal tube B. fenestrated tracheostomy tube C. oral pharyngeal airway D. oral endotracheal tube

B. fenesrated tracheostomy tube

An infant on a high frequency jet ventilator (rate of 150 b/m) has the following arterial blood gases: pH: 7.30 PaO2: 60 torr PaCO2: 50 torr HCO3-: 21 mEq/L Which of the following changes would best help to improve these results? A. Increase the inspiratory time B. Increase the drive pressure C. Increase the frequency to 190 br/min D. Increase the FIO2

B. increase the drive pressure. patient isn't ventilating but they are oxygenating. increasing pressure will increase VT = decrease PCO2

A 57-year-old post-op patient is receiving volume control ventilation. The respiratory therapist is having difficulty removing secretions when suctioning. The patient weighs 85 kg (187 lb), is orally intubated with a size 9.0 mm ID endotracheal tube, the vacuum level is set at 90 mm Hg and the suction catheter being used is a size 14 Fr. The therapist should A. switch to a larger catheter size. B. increase the vacuum level. C. switch to a Coude catheter. D. instill normal saline prior to suctioning.

B. increase the vacuum level

The peak inspiratory pressure on a pressure limited, time-cycled ventilator was changed from 20 cmH2O pressure to 25 cmH2O pressure. Which of the following alarms need to be adjusted at this time? A. PEEP B. low pressure C. peak pressure D. I:E ratio

B. low pressure alarm. if Leak in system won't get good ventilation. set 5-10 cmH20 below new PIP

A 2800 g neonate has just been orally intubated with a size 3.0 mm ID endotracheal tube. The respiratory therapist is preparing to suction the neonate and notices that there are size 6 French suction catheters at the bedside. The therapist should A. use the size 6 French catheters. B. obtain a size 4 French catheter. C. obtain a size 8 French catheter. D. obtain a size 10 French catheter.

B. obtain a size 4 French catheter ETT/2 x 3

A 4-year-old child has been brought to the emergency room with an acute infection, high fever, marked stridor and drooling. Which of the following should the therapist recommend at this time? A. Arterial blood gas B. Oral intubation C. Lateral neck X-ray D. 40% oxygen via cool mist tent

B. oral intubation "MARKED stridor"

A 44-year-old woman has just undergone a cholecystectomy. Over the last 48 hours the patient has the following laboratory findings: K+: 3 mEq/L Na+: 115 mEq/L Cl-: 80 mEq/L HCO3-: 24 mEq/L Urine output: 60 mL/hour BP: 125/95 mm Hg Based upon this information the respiratory therapist would conclude A. the patient is hyperkalemic B. the patient requires decreased fluid intake C. the patient is polycythemic D. the patient has a metabolic alkalosis

B. patient requires decreased fluid intake

An 18-month-old child admitted to the pediatric unit is playing with marbles. The child suddenly develops violent coughing and inspiratory stridor. The respiratory therapist has administered back blows and chest thrusts without success. Which of the following should the respiratory therapist now recommend? A.Chest compressions B.Rigid bronchoscopy C.Blind finger sweep D.Mouth to mouth resuscitation

B. rigid bronchoscopy

A 52-year-old, 5' 9" tall male patient who weighs 85 kg (187 lb) is being mechanically ventilated. An arterial blood gas has been obtained. Ventilatory data and blood gas results are below: Mode: VC, A/C FIO2: 0.40 Set rate: 10 br/min Total rate: 10 br/min VT: 550 mL PEEP: 5 cm H2O Mechanical VD: 50 mL pH: 7.28 PaCO2: 74 torr PaO2: 57 torr SaO2: 86% HCO3-: 23 mEq/L BE: 0 mEq/L The respiratory therapist should increase the A. VT to 700 mL. B. set rate to 14 br/min. C. FIO2 to 0.65. D. PEEP to 15 cm H2O.

B. set rate to 14 bpm

A respiratory therapist is performing spirometry on a patient with chronic bronchitis. Which of the following equipment would the respiratory therapist need? A. ergometer B. water seal spirometer C. pneumogram D. turbine pneumotachometer

B. water seal spirometer measuring volume directly

Which of the following would indicate that a patient is benefiting from PEP therapy? A. PaO2 increases B. Decreased work of breathing C. Breath sounds clear with coughing D. Minute ventilation decreases

Breath sounds clear when coughing

Which of the following would indicate that a patient is benefiting from PEP therapy? A. PaO2 increases B. Decreased work of breathing C. Breath sounds clear with coughing D. Minute ventilation decreases

Breath sounds clear with coughing. PEP therapy related to secretions

A respiratory therapist reviews a ventilator parameter sheet and finds that the peak inspiratory pressure has been gradually rising for the past several hours. Which of the following could be the cause for this change? 1. Bronchospasm 2. Increasing pulmonary compliance 3. Accumulation of secretions 4. Increasing airway resistance A. 2 only B. 1 & 3 only C. 1, 3 & 4 only D. 1, 2, 3 & 4

Bronchospasm, Secretions, Increasing Raw

What is the PAO2 for a patient breathing 30% oxygen at sea level? A. 100 - 105 torr B. 120 - 140 torr C. 155 - 170 torr D. 210 - 220 torr

C. 155 -170 torr PAO2= 7xFiO2% - PCO2

A patient in the intensive care unit has the following hemodynamic measurements: CVP: 6 mm Hg Mean PAP: 13 mm Hg PCWP: 7 mm Hg MAP: 86 mm Hg Cardiac output (QT): 4.0 L/min Cardiac index (QI): 2.1 L/min/m2 What is the systemic vascular resistance? A. 800 dynes/s/cm-5 B. 1300 dynes/s/cm-5 C. 1600 dynes/s/cm-5 D. 2400 dynes/s/cm-5

C. 1600 dynes SVR = MAP - CVP/ CO Convert to Dynes = x 80

What would be the oxygen consumption (VO2) for a patient with the following data: pH: 7.45 PaCO2: 39 torr PaO2: 94 torr HCO3-: 29 mEq/liter BE: +1 SaO2: 96% Hb: 15.8 gms QT: 5.2 LPM CaO2: 19.3 vol% CvO2: 14.3 vol% A. 150 mL/min B. 210 mL/min C. 260 mL/min D. 380 mL/min

C. 260 mL/min VO2= QT x (CaO2-CvO2) x 10

What is the normal range for the mean pulmonary artery pressure in an adult? A) 2-6 mm Hg B) 4-12 mm Hg C) 9-18 mm Hg D) 21-28 mm Hg

C. 9-18 mmHg

While reviewing a patient's medical record, the respiratory therapist notes that the patient has a Hb level of 6 g and a SpO2 of 94%. The patient is receiving oxygen by simple mask and is complaining of shortness of breath. The respiratory therapist would conclude that the patient has A. Hemophilia B. Hypernatremia C. Anemia D. Hypokalemia

C. Anemia

While reviewing a patient's medical record, the respiratory therapist notes that the patient has the following clinical data: pH: 7.45 PaCO2: 36 torr PaO2: 89 torr SpO2: 94% Na: 139 mEq/L K: 3.9 mEq/L Cl: 90 mEq/L Hb: 8 g/dL BUN: 15 mg/dL Glucose: 90 mg/dL The patient is receiving oxygen at 5 LPM via nasal cannula and is complaining of shortness of breath. The respiratory therapist would conclude that the patient has:

C. Anemia all values are normal but Hb

A 2-year-old child enters the emergency room. The mother states that the child was playing with friends and developed violent coughing and unilateral wheezing. Physical examination reveals a hyperresonant percussion note on the left and resonant percussion note on the right. Inspiratory and expiratory chest films indicate air trapping with no foreign bodies noted. The respiratory therapist should suspect that the child has A. pneumothorax. B. orthopnea C. aspirated a foreign object. D. tachyphylaxis.

C. Aspirated a foreign object-unilateral wheezing

A premature neonate with respiratory distress syndrome is being mechanically ventilated with a pressure-limited, time-cycled ventilator at the following settings: PIP: 25 cm H2O Rate: 24/min Flow: 7 L/min I time: 0.6 sec. FIO2: 0.45 PEEP: 4 cm H2O Arterial blood gas results from an umbilical artery line are as follows: pH: 7.29 PaCO2: 62 torr PaO2: 68 torr HCO3-: 22 mEq/L The therapist should now recommend: A. Increase PEEP B. Increase the I time C. Increase the rate D. Decrease PIP

C. Increase the rate, do this first then I-time if needed

An obese patient is being weaned from the ventilator following cholecystectomy. Arterial blood gas results reveal: pH: 7.40 PaO2: 96 torr PaCO2: 43 torr SaO2: 94% The patient is currently on: Tidal volume 650 mL, SIMV mandatory rate 4 /min, PEEP 5 cm H2O, and FIO2 0.35 even though she is not fully conscious. What should the respiratory therapist recommend at this time? A. Extubate B. Re-evaluate in 1 hour C. Place on a T-piece at a FIO2 0.45 D. Repeat arterial blood gases in 30 minute

C. Place on a T-piece at FiO2 .45. PT still unconscious-ventilator no longer necessary, but can't extubate.

While receiving postural drainage and percussion to her right lower lobe, a 44-year-old patient suddenly vomits and aspirates. The respiratory therapist should immediately A. place the patient in Fowlers position. B. encourage the patient to take deep breaths. C. suction the patient. D. administer acetylcysteine by SVN.

C. Suction the patient Do this first to remove vomit and prevent further aspiration

Which of the following will determine the aortic pulse pressure? A. systolic + diastolic + diastolic / 3 B. diastolic + pulse pressure / 3 C. systolic pressure - diastolic pressure D. stroke volume x heart rate x 10

C. Systolic pressure- diastolic pressure

An 1800 g neonate in the NICU is being monitored with a TcPO2 electrode. The TcPO2 electrode is reading 42 torr with the temperature set at 38oC. The PO2 from an umbilical artery sample is 72 torr. Which of the following would best explain the difference in these readings? A. There was an error in the arterial blood gas results. B. The TcPO2 electrode needs to be repositioned. C. The TcPO2 electrode temperature setting is too low. D. The TcPO2 electrode has been dislodged.

C. TcPO2 electrode temp setting is too low. Must be around 43-45 degrees C

A tracheostomy tube has just been changed on a patient receiving continuous ventilation. The patient suddenly becomes dyspneic and develops subcutaneous emphysema. This is most likely due to: A. Patient needs to be suctioned immediately B. Patient is having an allergic reaction to the Teflon tube C. Tracheostomy tube is malpositioned D. A larger tracheostomy tube needs to be inserted

C. Trach tube is malpositioned

A respiratory therapist is performing spirometry on a patient with chronic bronchitis. Which of the following equipment would produce an unreliable measurement? A. body plethysmograph B. water seal spirometer C. wright respirometer D. pressure differential pneumotachometer

C. Wright respirometer can't routinely recalibrate

Following indirect calorimetry, a 66-year-old patient's RQ is calculated to be 1.01. What food group is being primarily metabolized by this patient? A. Proteins B. Fats C. Carbohydrates D. Amnio acids

C. carbohydrates RQ factor: protein-0.82, fats-0.71

Mechanical Ventilation - A: Question 1 of 15 A 52-year-old, 5' 9" tall male patient weighing 85 kg (187 lb) is being mechanically ventilated. An arterial blood gas has been obtained. Ventilatory data and blood gas results are below: Mode: Assist/Control FIO2: 0.40 Mandatory rate: 10 Total rate: 10 VT: 650 mL PEEP: 5 cm H2O Mechanical VD: 100 mL pH: 7.28 PaCO2: 74 torr PaO2: 57 torr SaO2: 86% HCO3-: 23 mEq/L BE: 0 mEq/L Which of the following changes should be recommended at this time? A. Decrease the VT to 550 mL B. Decrease the mandatory rate to 8 C. Decrease the mechanical deadspace D. Decrease the FIO2 to 0.35

C. decrease the mechanical deadspace

A patient is on a continuous flow CPAP system. The respiratory therapist notes that the pressure drops to zero during inspiration. The therapist should: A. Initiate mechanical ventilation B. Change to an IPAP/EPAP system C. Increase the flow D. Decrease the threshold resistor

C. increase the flow

An adolescent patient admitted with a drug overdose is being mechanically ventilated at the following settings: Mode: PC, A/C Set Rate: 16 br/min Flow: 25 L/min Pressure limit: 40 cm H2O FIO2: 0.30 Exhaled VT: 500 mL The respiratory therapist observes wide fluctuations on the pressure manometer during inspiration. The most appropriate action would be to A. sedate the patient. B. increase the pressure limit. C. increase the flowrate. D. decrease the trigger setting.

C. increase the flowrate

Which of the following is the most appropriate technique for insertion of a nasopharyngeal airway? A. insert the airway in the opposite direction of its shape and rotate 180 degrees B. insert the airway in the opposite direction of its shape and rotate 90 degrees C. insert the airway in the same direction of its shape into the airway and do not rotate D. insert the airway in the same direction of its shape and rotate 180 degrees

C. insert airway in the same direction of its shape and do not rotate

During an incentive spirometry treatment using a flow-sensing device, the float will not rise during inspiration. This may be due to all of the following EXCEPT A. a crack in the casing. B. an obstructed mouthpiece. C. inspiratory effort is too strong. D. inspiratory flow is too slow.

C. inspiratory effort is too strong

While performing a routine ventilator check on a patient in the ICU, the respiratory therapist notes that the patient's cardiac monitor shows ventricular tachycardia at a rate of 165. The therapist determines the presence of a carotid pulse, although the patient is hypotensive and unresponsive. Which of the following should the respiratory therapist recommend? A. prepare for defibrillation B. administer a precordial thump C. prepare for synchronized cardioversion D. administer 1mg/kg of lidocaine

C. prepare for synchronized cardioversion

The respiratory therapist reviews the ventilator parameter sheet for a patient receiving mechanical ventilation. The peak and plateau pressure readings are as follows: PIP: 33,36,42,48,54 Pplat: 28,32,36,43,48 Based upon this information, the therapist should conclude that A. airway resistance is increasing. B. water is building up in the tubing. C. pulmonary compliance is decreasing. D. minute volume is increasing.

C. pulmonary compliance is decreasing. Raw isn't changing too much through hours(PIP-Pplat). Pplat keeps increasing because lungs Cst is decreasing

A patient in the intensive care unit has the following hemodynamic measurements: CVP: 9 mmHg Mean PAP: 24 mmHg PCWP: 9 mmHg MAP 93 mmHg Cardiac Output (QT): 5.4 L/min Cardiac index (QI): 2.9 Lmin/m2 A) right heart failure B) left heart failure C) pulmonary embolism D) normal cardiac fuction

C. pulmonary embolism CVP-High, Mean PAP-High(lungs), PCWP-Normal (rules out left heart problem), MAP-normal, QT-normal, QI-normal

A 32-week gestational age infant is receiving mechanical ventilation for hyaline membrane disease. The patient required a chest tube for a persistent pneumothorax. Two days later the chest radiograph reveals bilateral radiolucency, midline mediastinum, and the right hemidiaphragm slightly elevated. This would indicate A. atelectasis. B. bronchopulmonary dysplasia. C. fluid overload. D. resolution of a pneumothorax.

C. resolution of PNX

A patient is receiving oxygen via a Venturi mask at an FIO2 of 0.45. The nurse is complaining that the patient keeps removing the Venturi mask from his face. The respiratory therapist should do which of the following at this time? A. Tape the Venturi mask to the patient's face B. Restrain the patient C. Switch to a nasal cannula at 6 L/min. D. Intubate the patient orally

C. switch to a nasal cannula at 6 lpm

During a routine ventilator parameter check on a patient, you notice the low pressure alarm is being briefly activated prior to each breath. Which of the following is most likely the cause? A. the patient is fighting the ventilator B. the flow rate is set too fast C. the alarm delay is set too short D. the peak pressure limit is set too high

C. the alarm delay is set too short-disconnect alarm. if patient was fighting the ventilator or flow too fast-high alarm would go off

A fireman is brought into the emergency room after being pulled from a burning warehouse and is found to be unconscious. What is the best way to assess the oxygenation status of this patient? A. Arterial blood gas results B. Hb & Hct levels C. CO-oximetry results D. SpO2 monitoring

CO-oximetry results

Which of the following therapies would be most appropriate for a spontaneously breathing patient who has atelectasis with a low SpO2 on an FIO2 of 1.0? A. Intermittent Positive Pressure Breathing (IPPB) B. Continuous Positive Airway Pressure (CPAP) C. Continuous Mechanical Ventilation (CMV) D. Intrapulmonary Percussive Ventilation (IPV)

CPAP

The respiratory therapist has been paged to the ICU to assist in the treatment of a 98 kg (215 lb) man. The patient is pale, diaphoretic, and suddenly loses consciousness. No palpable pulse or blood pressure is measured. The ECG displays Normal sinus rhythm: The respiratory therapist should A. confirm the ECG in another lead. B. begin chest compressions. C. perform cardioversion. D. perform defibrillation.

CPR

A 10-year-old child with muscular dystrophy is receiving non-invasive ventilation at an IPAP of 10 cm H2O and an EPAP of 5 cm H2O. The set rate is 15 br/min. with the following arterial blood gases: pH: 7.34 PCO2: 52 torr PO2: 58 torr HCO3-: 26 mEq/L Which of the following should the respiratory therapist recommend? A. Switch to invasive ventilation. B. Decrease the set rate to 10 br/min. C. Change the IPAP to 15 cm H2O. D. Change the EPAP to 10 cm H2O.

Change the IPAP to 15 cmH2O

A 10-year-old child with muscular dystrophy is receiving non-invasive ventilation at an IPAP of 10 cm H2O and an EPAP of 5 cm H2O. The set rate is 15 br/min. with the following arterial blood gases: pH: 7.34 PCO2: 52 torr PO2: 58 torr HCO3-: 26 mEq/L Which of the following should the respiratory therapist recommend? A. Switch to invasive ventilation. B. Decrease the set rate to 10 br/min. C. Change the IPAP to 15 cm H2O. D. Change the EPAP to 10 cm H2O.

Change the IPAP to 15 cmH2O

A male patient (6' 2" tall, 80 kg, 176 lb) is being mechanically ventilated following an exploratory laporatomy at the following settings: Mode: VC, A/C Set rate: 12 /min. Total rate: 18 /min. FIO2: 0.50 VT: 700 mL PEEP: 6 cm H2O Arterial blood gas results are: pH: 7.49 PaCO2: 32 torr PaO2: 90 torr HCO3-: 24 mEq/L Based on this information, the respiratory therapist should A. add mechanical deadspace. B. change to VC, SIMV. C. decrease the FIO2 to 0.45. D. initiate pressure support.

Change to VC, SIMV

All of the following would be appropriate goals for a pulmonary rehabilitation program for a patient with COPD EXCEPT A. increased exercise tolerance. B. decreased hypoxic symptoms. C. improved resting blood gases. D. improved activities of daily living.

except for improved resting blood gases

A patient complaining of dyspnea has the following arterial blood gas results: pH: 7.36 PaCO2: 56 torr PaO2: 49 torr HCO3-: 34 mEq/L SaO2: 80% FIO2: .21 These results are best described as: A. acute respiratory acidosis B. acute metabolic alkalosis C. chronic respiratory acidosis D. chronic metabolic alkalosis

Chronic respiratory acidosis

A patient in the intensive care unit has the following hemodynamic measurements: CVP (mm Hg): 12 PAP (mm Hg): 48/25 PCWP (mm Hg): 18 MAP (mm Hg): 69 Cardiac output (L/min): 3.1 Cardiac index (L/min/m2): 1.7 These results are consistent with: A. Hypervolemia B. Congestive heart failure C. Pulmonary hypertension D. Cor pulmonale

Congestive Heart failure. CVP(Right heart-high), PAP(high), PCWP(before left heart-high), MAP(low)= left heart problems Fluid would affect all pressures.

A patient is receiving IPPB by mouthpiece. The therapist notes that the machine fails to cycle into the expiratory phase. The respiratory therapist should A. increase the flow. B. decrease the sensitivity. C. adjust the apnea timer. D. increase the pressure.

Increase the Flow. Pressure cycled machine. Causes to not reach pressure: Leak or flow

A patient is being mechanically ventilated in the intensive care unit. The following data is obtained: Mode: VC, SIMV Mandatory rate: 12 b/min Total rate: 18 b/min VT: 800 mL FIO2: 0.60 PIP: 31 cm H2O PEEP: 10 cm H2O pH: 7.41 PaCO2: 40 torr PaO2: 95 torr SaO2: 96% HCO3-: 23 mEq/L BE: +1 mEq/L PECO2: 30 torr Hb: 15 g/dL What should the respiratory therapist report as the CaO2? A. 18.40 vol% B. 18.60 vol% C. 19.30 vol% D. 19.59 vol%

D. 19.59 vol% (HB x 1.34 x SaO2) + (PaO2 x .003)

A 70 kg (154 lb) male is receiving non-invasive positive pressure ventilation (NIPPV) with a BiPAP ventilator set to 15 / 5. Arterial blood gases are as follows: pH: 7.33 PaCO2: 49 torr PaO2: 83 torr Which of the following IPAP / EPAP settings would best improve the patient's condition? A. 10 / 2 B. 15 / 10 C. 20 / 10 D. 20 / 5

D. 20/5

What is the normal range for the pulmonary artery systolic pressure in an adult? A. 2-6 mm Hg B. 4-12 mm Hg C. 9-18 mm Hg D. 21-28 mm Hg

D. 21-28 mmHg Normal PAP 25/8 = 8+8+25/3

The following data was collected for a patient in the intensive care unit: pH: 7.38 PaCO2: 40 torr PaO2: 90 torr HCO3-: 25 mEq/L BE: +1 mEq/L SaO2: 98% FIO2: .30 Hb: 15g/dl RER: 0.8 PB: 747 torr PECO2: 10 torr What should be reported as the patient's VD/VT ratio? A. 20% B. 35% C. 50% D. 75%

D. 75%

An air flowmeter and an oxygen flowmeter are being used to deliver 40% oxygen to a patient via a non-rebreathing mask. A total flow of 12 LPM is required to prevent the non-rebreathing bag from deflating. How many liters of air and how many liters of oxygen should the therapist use? A. 2 LPM air, 6 LPM oxygen B. LPM air, 8 LPM oxygen C. 6 LPM air, 6 LPM oxygen D. 9 LPM air, 3 LPM oxygen

D. 9 LPM air, 3 LPM oxygen 3:1 ratio

A male infant born at 36 weeks gestation was delivered by Cesarean section. The newborn is exhibiting signs of respiratory distress including nasal flaring and mild retractions. Physical examination reveals HR 160/min, RR 52/min, BP 60/40 mm Hg. The chest radiograph indicates some scattered densities. The respiratory therapist should recommend A. nasal CPAP at 5 cm H2O. B. BiPAP ventilation. C. 40% oxygen via hood. D. arterial blood gases.

D. ABG. Vitals are normal. Infant not that bad. If they did have RDS then other options appropriate

A 27-week gestation age neonate with respiratory distress syndrome is receiving high frequency oscillatory ventilation at the following settings: PIP: 20 cm H2O Frequency: 12 Hz I time: 30% FIO2: 0.55 PEEP: 4 cm H2O pH: 7.15 PaCO2: 62 torr PaO2: 46 torr HCO3-: 22 mEq/L The respiratory therapist should increase the A. PEEP. B. FIO2. C. frequency. D. amplitude.

D. Amplitude. Increase = Increase Vt Decrease Frequency =Decrease Vt

A post-operative craniotomy patient was thrashing around while on a volume-cycled microprocessor ventilator. The physician has asked for your recommendation in the management of this patient to prevent him from harming himself and to also stabilize his ICP. Which of the following drugs would you recommend administering at this time? A. Anectine B. Versed C. Valium D. Morphine

D. Morphine. Sedates, relieves pain and controls ventilator pain. Anectine-short paralyzing, Versed-anti-anxiety, Valium-sedates

A patient in the emergency department has frothy secretions, moist crackles, and tachypnea. The patient has marked dyspnea and a history of heart disease. Which of the following should the respiratory therapist recommend? 1. Suction immediately 2. Administer 100% oxygen 3. Place in Fowlers position 4. Administer furosemide A.1, 2 & 3 only B.1, 3 & 4 only C.1, 2 & 4 only D.2, 3, & 4 only

D. O2, fowlers, furosemide PT has severe pulmonary edema treat it aggressively SXN only delays good therapy

A patient in the intensive care unit has the following hemodynamic measurements: CVP: 4 mm Hg PAP: 48/16 mm Hg PCWP: 8 mm Hg MAP: 92 mm Hg Cardiac Output: 5 L/min. Cardiac Index: 2.5 L/min/m2 The respiratory therapist should recommend the administration of A.bumex. B.dopamine. C.lidocaine. D.oxygen.

D. Oxygen (vasodilation decreasing pressures) Bumex (diuretic), dopamine(vasopresser for shock), lidocaine(anti-arrhythmic PVC)

Shortly after extubating a patient, the therapist notes an increase in the patient's work of breathing, intercostal retractions, marked stridor and a decreased SpO2. The therapist should recommend: A. Cool aerosol therapy with 50% oxygen B. Assisted ventilation with a resuscitation bag and mask C. CPAP therapy with 40% oxygen D. Reintubate orally

D. Reintubate orally

When performing a bronchoscopy on an intubated patient, which of the following CANNOT be visualized? A. Trachea B. Bronchus intermedius C. Carina D. Vocal cords

D. Vocal Cords proximal to trachea because patient is intubated

A three day postoperative laryngectomy patient is receiving 50% oxygen with a cool aerosol via a tracheostomy collar. The secretions being suctioned by the therapist are thick and difficult to aspirate. The therapist should recommend: A. Increase the frequency of suctioning B. Instill 2 mL of normal saline prior to suctioning C. Decrease the oxygen to 45% D. Add a heating element to the aerosol

D. add a heating element to the aerosol

A "broken" flow/volume loop and/or volume/pressure loop will occur with: A. airway resistance B. overdistension C. auto PEEP D. airway leak

D. airway leak

A 80 kg (176 lb) male suffering from acute respiratory distress syndrome is being mechanically ventilated at the following settings: Mode: VC, A/C Tidal Volume: 500 mL Set Rate: 10 br/min Total Rate: 10 br/min PIP: 68 cm H2O FIO2: 0.45 PEEP: 8 cm H2O The following patient information is available: pH: 7.37 PaCO2: 44 torr PaO2: 89 torr HCO3-: 22 mEq/L The respiratory therapist should A. decrease PEEP. B. change to SIMV. C. add pressure support. D. change to pressure control.

D. change to pressure control, to control PIP < 50. PT not spontaneously breathing so B/C don't work. PEEP already <10

The pulmonary physician has made his patient rounds in the intensive care unit. The physician has remarked in the progress notes that he would like to make a change to the patient's ventilator settings. The therapist's most appropriate action should be to: A. initiate the ventilator changes B. write a verbal order for the change C. ask the nurse to write the order D. contact the physician to write the order

D. contact the physician to write the order

A 78-year-old male patient is being treated for obstructive sleep apnea with bilevel positive airway pressure at night. Arterial blood gas results reveal an elevated PaCO2 level. The respiratory therapist should recommend increasing the? A. oxygen concentration. B. expiratory positive airway pressure (EPAP). C. inspiratory flow rate. D. inspiratory positive airway pressure (IPAP).

D. increase IPAP

A 26-year-old patient with shortness of breath is admitted to the emergency room. The patient states that he was running in Central Park with a friend and could not catch his breath. Bedside assessment reveals the following data: Pulse: 45 Respirations: 50 br/min Color: cyanotic SpO2: 68% on room air Breath sounds: absent on the right Based upon the above information the respiratory therapist should recommend: A. request a STAT chest x-ray. B. administer 30% oxygen. C. administer IPPB stat. D. insert a chest tube into the 2nd intercostal space on the right side in the midclavicular line.

D. insert a chest tube into the 2nd intercostal space on the right side in the midclavicular line.

A 44-year-old woman has just undergone a cholecystectomy. Over the last 48 hours the patient has the following laboratory findings: K+: 2 mEq/L Na+: 135 mEq/L Cl-: 100 mEq/L HCO3-: 32 mEq/L Urine output: 40 mL/hour Based upon this information, the respiratory therapist would conclude that the patient

D. metabolic alkalosis K(low), Na(normal), Cl(normal), HCO3(high) confirming answer

A patient is on a ventilator and is in the process of being weaned. What is the best way to continuously monitor the minute ventilation? A. Chest transducers B. Thermistors C. Water seal spirometer D. Pneumotachometer

D. pneumotachomter. measure flow and converts to volume breath by breath. Water seal spirometer is not good for continuous measurements

A 2-month-old infant is receiving 40% oxygen via an oxygen hood. While performing oxygen rounds, the respiratory therapist analyzes the FIO2 inside the hood and discovers that the FIO2 is registering 0.50. The respiratory therapist calibrated the oxygen analyzer at the beginning of the shift. The respiratory therapist should A. check to see that the hood is sealed tightly around the infant's neck. B. decrease the flow to the hood. C. check to see that the temperature setting on the humidifier is at 34oC. D. recalibrate the oxygen analyzer.

D. recalibrate the oxygen analyzer leak would decrease the FiO2 not increase

A 5' 10", 80 kg (176 lbs) male patient with multiple trauma from a motorcycle accident has developed ARDS and is being ventilated at the following settings: Mode: VC, SIMV VT: 500 mL Set Rate: 14 br/min. FIO2: 0.60 PEEP: 12 cm H2O Arterial blood gas results reveal: pH: 7.35 PaO2: 72 torr PaCO2: 44 torr SpO2: 93% The patient is conscious and pulling on the IV lines and ventilator tubing. At this time, the respiratory therapist should recommend? A. changing to assist/control mode. B. restraining the patient. C. increasing the set rate to 16 br/min. D. sedating the patient.

D. sedating the patient

All of the following will affect the accuracy of a capnograph EXCEPT A) long sampling line B) low sampling flow C) condensation in the tubing D) use of a desiccant

D. use of a desiccant-chemical that removes moisture from the gas creating a more accurate measurement Long sampling line will take longer for gas to pass thru to reach analyzer, low sampling flow wont give enough info for good reading, condensation in the tubing can cause obstruction in line

A 52-year-old, 6' 1" tall male patient weighs 80 kg (176 lb) and is being mechanically ventilated. Ventilatory data and blood gas results are below: Mode: VC, A/C FIO2: 0.40 Set rate: 14 br/min Total rate: 14 br/min VT: 750 mL PEEP: 10 cm H2O pH: 7.49 PaCO2: 28 torr PaO2: 87 torr SaO2: 96% HCO3-: 25 mEq/L BE: +1 mEq/L Which of the following changes should be recommended at this time? A. Decrease the set rate to 10 br/min. B. Decrease the VT to 350 mL. C. Increase the PEEP to 15 cm H2O. D. Add 50 mL of mechanical deadspace.

Decrease the set RR. Do smaller adjustments over large (Vt dropped by 400)

A 52-year-old, 6' 1" tall male patient weighs 80 kg (176 lb) and is being mechanically ventilated. Ventilatory data and blood gas results are below: Mode: VC, A/C FIO2: 0.40 Set rate: 14 br/min Total rate: 14 br/min VT: 750 mL PEEP: 10 cm H2O pH: 7.49 PaCO2: 28 torr PaO2: 87 torr SaO2: 96% HCO3-: 25 mEq/L BE: +1 mEq/L Which of the following changes should be recommended at this time? A. Decrease the set rate to 10 br/min. B. Decrease the VT to 350 mL. C. Increase the PEEP to 15 cm H2O. D. Add 50 mL of mechanical deadspace.

Decrease the set rate to 10. Don't want to drop Vt so drastically to 350 mL, that's why we picked RR

A patient's PvO2 has decreased from 30 torr to 20 torr. This is indicative of a/an: A. Decreased SVR B. Increased cardiac index C. Decreased cardiac output D. Decreased PVR

Decreased Cardiac Output. Heart is represented by venous blood samples.

A patient in the intensive care unit is suffering left heart failure. Which of the following drugs will increase the strength of contraction and improve cardiac output? A. digitalis B. atropine C. isuprel D. lidocaine

Digitalis. Atropine/Isuprel-bradycardia, lidocaine-PVC

A patient in the intensive care unit is suffering left heart failure. Which of the following drugs will increase the strength of contraction and improve cardiac output? A. digitalis B. atropine C. isuprel D. lidocaine

Digitalis> increases CO Atropine used for bradycardia

A 48-year-old patient is receiving mechanical ventilation following a hernia repair. He is 6' tall and weighs 75 kg (165 lb). Prior to initiating a spontaneous breathing trial, the respiratory therapist notes the following information: FIO2: 0.35 SIMV mandatory rate: 6 /min. Total rate: 14 /min. VT: 600 mL MIP: -26 cm H2O Spont. VT: 400 mL pH: 7.39 PaCO2: 42 torr PaO2: 88 torr HCO3-: 23 mEq/L The patient is to be placed on CPAP with an FIO2 of 0.40. Five minutes into the SBT, his respiratory rate increases to 28 /min., heart rate changes from 110 /min. to 135 /min. and blood pressure changes from 112/70 mm Hg to 140/88 mm Hg. At this time, the therapist should A. shorten the SBT by 5 minutes. B. continue the SBT. C. discontinue the SBT. D. increase the FIO2 to 0.45.

Discontinue the SBT. HR increases > 20bpm is adverse reaction

A premature baby is receiving an FIO2 of .40 and CPAP at 5 cmH2O. As the respiratory therapist increases the CPAP to 7 cmH2O, the baby's respiratory rate increases to 58 per minute and the TcPCO2 reading increases with a stable SpO2 monitor reading. The respiratory therapist should recommend which of the following? A. Discontinue the CPAP B. Draw an arterial blood gas sample C. Increase the CPAP to 10 cmH2O D. Place the baby in an oxyhood at an FIO2 0.45

Draw an ABG

A premature baby is receiving an FIO2 of .40 and CPAP at 5 cmH2O. As the respiratory therapist increases the CPAP to 7 cmH2O, the baby's respiratory rate increases to 58 per minute and the TcPCO2 reading increases with a stable SpO2 monitor reading. The respiratory therapist should recommend which of the following? A. Discontinue the CPAP B. Draw an arterial blood gas sample C. Increase the CPAP to 10 cmH2O D. Place the baby in an oxyhood at an FIO2 0.45

Drawn an ABG

A post-operative craniotomy patient is receiving mechanical ventilation and has an increased intracranial pressure reading. The goals of mechanical ventilation for this patient include all of the following EXCEPT A. increased PaCO2. B. keep airway pressure to a minimum. C. hyperoxygenate. D. use minimum PEEP levels.

EXCEPT: Increased PaCO2

Which of the following items should the respiratory therapist select prior to performing orotracheal intubation? 1. Various sizes of endotracheal tubes 2. Water-soluble lubricant 3. Stylet 4. Magill forceps A. 1 and 2 only B. 2 and 4 only C. 1 and 3 only D. 3 and 4 only

ETT tubes and Stylet

A 66-year-old patient with tuberculosis treated with antibiotics is no longer considered contagious and is ready for discharge from the hospital. During his hospitalization, he received IPPB therapy using a Bird Mark VII ventilator. What would be the most appropriate method of sterilizing the Bird Mark VII? A. Steam autoclave B. Ethylene oxide C. Alkaline glutaraldehyde D. Pasteurization

Ethylene Oxide-$$, low temps and no solution Steam autoclave-high temps will melt the mask

A 66-year-old patient with tuberculosis treated with antibiotics is no longer considered contagious and is ready for discharge from the hospital. During his hospitalization, he received IPPB therapy using a Bird Mark VII ventilator. What would be the most appropriate method of sterilizing the Bird Mark VII? A. Steam autoclave B. Ethylene oxide C. Alkaline glutaraldehyde D. Pasteurization

Ethylene oxide

During cardiopulmonary resuscitation of a 78-year-old patient, the respiratory therapist provides ventilation with a manual resuscitation bag attached to the patient's endotracheal tube. While squeezing the bag, the therapist observes inadequate chest movement. Which of the following is the LEAST likely cause? A. Missing inlet valve B. Defective endotracheal tube cuff C. Incorrect tube placement D.Excessive oxygen flow

Excessive O2 flow

A 40-year-old patient who smokes 2 packs of cigarettes per day has a carboxyhemoglobin level of 6.4%. These results are most consistent with A. Severe COPD B. History of dyspnea on exertion C. Need for supplemental oxygen D. Expected level for this patient

Expected level for this patient

A 45-year-old female patient who weighs 60 kg (132 lb) remains intubated in the recovery room after an exploratory laparoscopy. She has been receiving 6 cm H2O of CPAP with an FIO2 of 0.35 for the past 2 hours. Arterial blood gases and vital signs are stable with the following information obtained: RR: 16 /min. MIP: -36 cm H2O VT: 400 mL VC: 1.2 L Based upon this information, the respiratory therapist should recommend A. maintaining the current therapy. B. decreasing the CPAP to 4 cm H2O. C. extubating and monitoring the patient. D. obtaining a portable chest x-ray.

Extubate and monitor the patient

A 45-year-old female patient who weighs 60 kg (132 lb) remains intubated in the recovery room after an exploratory laparoscopy. She has been receiving 6 cm H2O of CPAP with an FIO2 of 0.35 for the past 2 hours. Arterial blood gases and vital signs are stable with the following information obtained: RR: 16 /min. MIP: -36 cm H2O VT: 400 mL VC: 1.2 L Based upon this information, the respiratory therapist should recommend A. maintaining the current therapy. B. decreasing the CPAP to 4 cm H2O. C. extubating and monitoring the patient. D. obtaining a portable chest x-ray.

Extubate the patient

In order to monitor compliance of hospital employees in a smoking cessation program, the respiratory therapist should monitor the employees' A. PaO2. B. FECO. C. PETCO2. D. FENO.

FECO

In order to monitor compliance of hospital employees in a smoking cessation program, the respiratory therapist should monitor the employees' A. PaO2. B. FECO. C. PETCO2. D. FENO.

FECo-exhaled carbon monoxide

Which of the following would be the best test to evaluate a patient's partial vocal cord paralysis? A. Flow volume loop B. MVV C. SB nitrogen elimination D. DCO

Flow volume loop

A patient complains of shortness of breath while receiving oxygen via transtracheal catheter. Her pulse oximeter reading has decreased from 92% to 85%. The respiratory therapist should first A. increase the flow to the catheter. B. flush the catheter with saline. C.administer metaproterenol via small volume nebulizer. D.replace the transtracheal catheter with a nasal cannula.

Flush the catheter with saline

A patient complains of shortness of breath while receiving oxygen via transtracheal catheter. Her pulse oximeter reading has decreased from 92% to 85%. The respiratory therapist should first A. increase the flow to the catheter. B. flush the catheter with saline. C. administer metaproterenol via small volume nebulizer. D. Replace the transtracheal catheter with a nasal cannula.

Flush the catheter with saline, patient may have a mucous plug

A patient in the intensive care unit has the following lab results: pH: 7.41 PaCO2: 39 torr PaO2: 96 torr HCO3-: 24 mEq/L SaO2: 97% Hb: 8 g/dL Which of the following would have the greatest impact on this patient's ability to deliver oxygen to the body tissue? A. pH B. PaO2 C. SaO2 D. Hb

Hb

A tympanic percussion note is usually present in which of the following conditions? 1. gastric distension 2. pleural effusion 3. pneumothorax 4. endocarditis A. 1 & 3 only B. 2 & 4 only C. 3 & 4 only D. 1, 2 & 3 only

Gastric distension and PNX

A 32-year-old victim of a motor vehicle accident (weight 80 kg, 176 lb) is on mechanical ventilation at the following settings: Tidal volume 700 mL, SIMV mandatory rate 10/min, and FIO2 0.65. As the PEEP level was increased from 17 cm H2O to 20 cm H2O, the respiratory therapist observed that the heart rate changed from 128/min to 106/min. At this time, the therapist should recommend A. inserting a balloon tipped flow directed catheter. B. drawing an arterial blood gas. C. reassessing the patient in 20 minutes. D. decreasing the PEEP.

Get an ABG

A 32-year-old victim of a motor vehicle accident (weight 80 kg, 176 lb) is on mechanical ventilation at the following settings: Tidal volume 700 mL, SIMV mandatory rate 10/min, and FIO2 0.65. As the PEEP level was increased from 17 cm H2O to 20 cm H2O, the respiratory therapist observed that the heart rate changed from 128/min to 106/min. At this time, the therapist should recommend A. inserting a balloon tipped flow directed catheter. B. drawing an arterial blood gas. C. reassessing the patient in 20 minutes. D. decreasing the PEEP.

Get an ABG

Which of the following devices would provide adequate humidity with minimal condensation in the ventilator tubing? A. heated cascade humidifier B. heated wick humidifier C. heat moisture exchanger D. ultrasonic nebulizer

HME. up next to ETT trapping exhaled gas to humidify gas to patient

A 47-year-old patient admitted for sepsis has a CaO2 value of 12.5 vol%. The patient does not appear cyanotic. Which of the following would be the most important to further evaluate the patient's oxygenation status? A. PaO2 B. SaO2 C. Hb D. PAO2

Hb

A patient in the intensive care unit has the following lab results: pH: 7.41 PaCO2: 39 torr PaO2: 96 torr HCO3-: 24 mEq/L SaO2: 97% Hb: 8 g/dL Which of the following would have the greatest impact on this patient's ability to deliver oxygen to the body tissue? A. pH B. PaO2 C. SaO2 D. Hb

Hb

A patient in the intensive care unit has the following hemodynamic measurements: CVP (mm Hg): 12 PAP (mm Hg): 48/25 PCWP (mm Hg): 18 MAP (mm Hg): 104 Cardiac output (L/min): 7.1 Cardiac index (L/min/m2): 3.7 These results are consistent with: A. Hypervolemia B. Congestive heart failure C. Pulmonary hypertension D. Cor pulmonale

Hypervolemia

A patient in the intensive care unit has the following hemodynamic measurements CVP (mm Hg): 12 PAP (mm Hg): 48/25 PCWP (mm Hg): 18 MAP (mm Hg): 104 Cardiac output (L/min): 7.1 Cardiac index (L/min/m2): 3.7 These results are consistent with: A. Hypervolemia B. Congestive heart failure C. Pulmonary hypertension D. Cor pulmonale

Hypervolemia.

A multiple trauma victim with internal hemorrhage is being monitored via pulse oximetry. Which of the following conditions would affect the accuracy of her SpO2 readings? A. hypotension B. hyperoxia C. hypocarbia D. 1yperthermia

Hypotension

The respiratory therapist is assessing a patient's vital signs and notes that the pulse feels weak and thready. This would most likely be associated with which of the following conditions? A. hypoxemia B. hypovolemia C. increase cardiac index D. increase systemic vascular resistance

Hypovolemia

The respiratory therapist is assessing a patient's vital signs and notes that the pulse feels weak and thready. This would most likely be associated with which of the following conditions? A. hypoxemia B. hypovolemia C. increase cardiac index D. increase systemic vascular resistance

Hypovolemia

A 3-year-old child with cystic fibrosis is being mechanically ventilated at the following settings: PIP: 34 cm H2O I time: 1.0 second Rate: 22 /min. FIO2: 0.60 PEEP: 6 cm H2O Mode: PC, SIMV pH: 7.36 PaCO2: 44 torr PaO2: 49 torr HCO3-: 24 mEq/L The respiratory therapist should adjust the A. PEEP to 8 cm H2O. B. FIO2 to 0.65. C. set rate to 24 /min. D. PIP to 36 cm H2O.

Increase PEEP to 8 cmH2O

The respiratory therapist reviews the ventilator flow sheet for a post-op patient. The ventilator settings have not been changed during the past 24 hours. The peak and plateau pressures were recorded as follows: PIP: 28,35,50 Pplat: 23,30,44 PEEP: 5,5,5 This information would indicate that A. airway resistance is increasing. B. lung compliance is increasing. C. the patient can begin weaning. D. the PEEP should be increased.

Increase PEEP. PIP and Pplat rising but difference staying the same=Raw not changing. Static compliance is decreasing-Tx=increase PEEP to open alveoli

A premature neonate with respiratory distress syndrome is being mechanically ventilated with a pressure-limited, time-cycled ventilator at the following settings: PIP: 25 cm H2O Rate: 28/min Flow: 7 L/min I time: 0.5 sec. FIO2: 0.70 PEEP: 4 cm H2O pH: 7.30 PaCO2: 59 torr PaO2: 66 torr HCO3-: 22 mEq/L The therapist should now recommend: A. Increase PEEP B. Increase the I time C. Decrease the rate D. Decrease PIP

Increase Ti.

When expiratory flow doesn't return to baseline we need to?

Increase flow to increase Te to prevent Auto PEEP

An 1800 g neonate is receiving 30% oxygen via an oxyhood with the following arterial blood gas results obtained: pH: 7.34 PaO2: 46 torr PaCO2: 47 torr HCO3-: 22 mEq/L Based upon this information, the respiratory therapist should recommend which of the following at this time? A. Intubate/initiate mechanical ventilation with an FIO2 0.55 B. Increase the FIO2 to 0.35 C. Administer 5 cm H2O endotracheal tube CPAP with an FIO2 0.60 D. Increase the FIO2 to 0.50

Increase the FiO2 to 50%. 35% isn't a big enough increase

A 5' 2", 65 kg (143 lb) female patient with a sub-arachnoid hemorrhage is being mechanically ventilated at the following settings: Mode: VC, A/C VT: 525 mL Set rate: 12 br/min Total rate: 12 br/min FIO2: .40 PEEP: 0 I:E Ratio: 1:2 The following information is available: pH: 7.35 PaCO2: 41 torr PaO2: 85 torr HCO3-: 23 mEq/L SaO2: 94% Intracranial Pressure: 30 mm Hg The respiratory therapist should A. increase the tidal volume. B. increase the respiratory rate. C. add 50 ml of deadspace. D. add expiratory resistance.

Increase the RR rate. Keep ICP < 10mmHg. Fix=hyperventilate, Vt already on high end

A 5' 2", 65 kg (143 lb) female patient with a sub-arachnoid hemorrhage is being mechanically ventilated at the following settings: Mode: VC, A/C VT: 525 mL Set rate: 12 br/min Total rate: 12 br/min FIO2: .40 PEEP: 0 I:E Ratio: 1:2 The following information is available: pH: 7.35 PaCO2: 41 torr PaO2: 85 torr HCO3-: 23 mEq/L SaO2: 94% Intracranial Pressure: 30 mm Hg The respiratory therapist should A. increase the tidal volume. B. increase the respiratory rate. C. add 50 ml of deadspace. D. add expiratory resistance.

Increase the RR. want patient to hyperventilate. Vt already on high end so we increase RR

A patient is receiving IPPB by mouthpiece. The therapist notes that the machine fails to cycle into the expiratory phase. The respiratory therapist should A.increase the flow. B. decrease the sensitivity. C. adjust the apnea timer. D. increase the pressure.

Increase the flow

A patient is receiving CPAP therapy and the pressure is fluctuating between +5 and -8 cmH2O. What should the therapist do to stabilize the CPAP therapy? A. increase the flowrate B. decrease the pressure C. check for system leaks D. check for sticking valves

Increase the flowrate

A post-operative craniotomy patient is receiving mechanical ventilation and has an increased intracranial pressure reading. The goals of mechanical ventilation for this patient include all of the following EXCEPT A. increased PaCO2. B. keep airway pressure to a minimum. C. hyperoxygenate. D. use minimum PEEP levels.

Increased PaCO2

A 57-year-old male patient is being mechanically ventilated following removal of his right upper lobe. The chest radiograph shows right side consolidation. An arterial blood gas has been obtained. Ventilatory data and blood gas results are: Mode: VC, SIMV FIO2: 1.0 Set rate: 20 br/min Total rate: 20 br/min VT: 450 mL PIP: 55 cm H2O PEEP: 18 cm H2O pH: 7.41 PaCO2: 39 torr PaO2: 43 torr HCO3: 23 mEq/L BE: 0 mEq/L The respiratory therapist should recommend A. independent lung ventilation. B. inverse ratio ventilation. C. pressure support ventilation. D. pressure control ventilation.

Independent lung ventilation

A patient is suspected of having a pleural effusion. Before performing a thoracentesis, what should the therapist recommend? A. lateral decubitus chest radiograph B. arterial blood gas analysis C. ventilation and perfusion scans D. flexible bronchoscopy

Lateral Decubitus chest radiograph

A patient is suspected of having a pleural effusion. Before performing a thoracentesis, what should the therapist recommend? A. lateral decubitus chest radiograph B. arterial blood gas analysis C. ventilation and perfusion scans D. flexible bronchoscopy

Lateral decubitus chest radiograph

A patient is being discharged from the hospital. The physician has ordered portable oxygen therapy at home. The patient has been instructed to use the oxygen at 1-2 L/min during the day and PRN at night. Which of the following should the respiratory therapist recommend? A. E cylinders B. Concentrator C. Liquid system D. Molecular sieve

Liquid O2 is most portable

A patient is being discharged from the hospital. The physician has ordered portable oxygen therapy at home. The patient has been instructed to use the oxygen at 1-2 L/min during the day and PRN at night. Which of the following should the respiratory therapist recommend? A. E cylinders B. Concentrator C. Liquid system D. Molecular sieve

Liquid system-regular use and portable

A 15-year-old female patient who weighs 55 kg (121 lb) was admitted to the ICU with multiple trauma from a four-wheeler accident and has developed ARDS. The respiratory therapist notes the following data: Mode: VC, SIMV Set rate: 12 br/min. Total rate: 14 br/min. FIO2: 0.60 PEEP: 10 cm H2O VT: 350 mL pH: 7.34 PCO2: 38 torr PO2: 79 torr HCO3-: 21 mEq/L She appears agitated, is using her accessory muscles, and is attempting to pull out her IV lines. Which of the following should the therapist recommend at this time? A. Change to VC, A/C B. Pancuronium bromide C. Succinylcholine D. Lorazepam

Lorazepam-sedation Pancuronium-paralytic Succinylcholine- short lasting paralytic

A 15-year-old female patient who weighs 55 kg (121 lb) was admitted to the ICU with multiple trauma from a four-wheeler accident and has developed ARDS. The respiratory therapist notes the following data: Mode: VC, SIMV Set rate: 12 br/min. Total rate: 14 br/min. FIO2: 0.60 PEEP: 10 cm H2O VT: 350 mL The arterial blood gases are as follows: pH: 7.34 PCO2: 38 torr PO2: 79 torr HCO3-: 21 mEq/L She appears agitated, is using her accessory muscles, and is attempting to pull out her IV lines. Which of the following should the therapist recommend at this time? A. Change to VC, A/C B. Pancuronium bromide C. Succinylcholine D. Lorazepam

Lorezapam -sedation

An air/oxygen proportioner is used to provide an FIO2 of 0.55 by a non-rebreather mask. The blender alarm is sounding. Which of the following is the most likely cause? A. Low oxygen inlet pressure B. High air inlet pressure C. Faulty humidifier bottle connection D. Excessive flow to the non-rebreather mask

Low O2 inlet pressure. adequate pressure from both sides of blender means no alarm

A 6' 2" tall, 80 kg (176 lb) male patient with alpha 1 protease inhibitor deficiency is being mechanically ventilated at the following settings: Mode: VC, SIMV Set rate: 12 br/min. Total rate: 16 br/min. Tidal volume (set): 650 mL Tidal volume (spontaneous): 320 mL Peak flow: 55 L/min FIO2: 0.30 His vital signs are stable and he is sleeping comfortably. An arterial blood gas has been drawn with the following results: pH: 7.37 PaCO2: 51 torr PaO2: 68 torr HCO3-: 29 mEq/L Which of the following should the respiratory therapist recommend? A. Increase the set rate B. Decrease the tidal volume C. Increase the FIO2 D. Maintain current settings

Maintain current settings

A 75 kg (165 lb) male patient is being mechanically ventilated following coronary artery bypass surgery. He is currently on the following settings: Mode: VC, SIMV Set rate: 10 br/min. Total rate: 14 br/min. FIO2: 0.40 VT: 650 mL PEEP: 8 cmH2O Arterial blood gas results show: pH: 7.41 PCO2: 44 torr PO2: 85 torr HCO3-: 23 mEq/L You would recommend which of the following at this time? A. Decrease the FIO2. B. Decrease the set rate to 8 br/min. C. Increase the PEEP to 10 cmH2O. D. Maintain the current settings.

Maintain current settings

A 52-year-old, 5' 9" tall male patient weighing 85 kg (187 lb) is being mechanically ventilated. An arterial blood gas has been obtained. Ventilatory data and blood gas results are below Mode: VC, A/C FIO2: 0.40 Mandatory rate: 16 /min Total rate: 16 /min VT: 800 mL PEEP: 5 cm H2O pH: 7.47 PaCO2: 31 torr PaO2: 87 torr SaO2: 96% HCO3-: 23 mEq/L BE: 0 mEq/L Which of the following changes should be recommended at this time? A. Increase the rate to 20 /min B. Increase the PEEP to 10 cm H2O C. Add 200 mL of mechanical deadspace D. Maintain current settings

Maintain the current settings

While reviewing a patient's medical record the respiratory therapist notes that the patient has a Hb level of 14g/dL and an SpO2 of 94%. The patient is receiving oxygen by simple mask at 6 L/min. The respiratory therapist should recommend: A. Non-rebreather mask at 10 LPM B. A unit of whole blood C. CPAP therapy D. Maintain current therapy

Maintain the current therapy

Which of the following is a potential side effect of inhaled nitric oxide therapy? A. Systemic hypotension B. Formation of nitrous oxide C. Oxygen toxicity D. Methemoglobinemia

Methemoglobinemia

A 5-year-old patient requires low-flow oxygen therapy. Which of the following devices should the respiratory therapist recommend? A. Oxygen tent B. Simple oxygen mask C. Oxygen hood D. Nasal cannula

Nasal Cannula

A 5-year-old patient requires low-flow oxygen therapy. Which of the following devices should the respiratory therapist recommend? A. Oxygen tent B. Simple oxygen mask C. Oxygen hood D. Nasal cannula

Nasal Cannula

A physician asks the respiratory therapist to deliver a precise FIO2 of 0.45 to a patient. Which of the following should the therapist recommend? A. Nasal cannula at 6 L/min B. Simple mask at 5 L/min C. Partial rebreather mask at 8 L/min D. Non-rebreathing mask and blender

Non-rebreathing mask and blender. The only high flow delivery option

Pre- and post-bronchodilator spirometry study is performed on a patient with the following results: Pre FVC-89, Pre FEV1-81 Post FVC-90, Post FEV1-82 These results can best be described as: A. normal spirometry. B. obstructive only. C. restrictive only. D. both obstructive and restrictive.

Normal spirometry

After injecting a small amount of air into the balloon of a pulmonary artery catheter, the respiratory therapist sees a small amplitude change with the mean pressure reading 2 points below the PA end-diastolic pressure. Based upon this information, the therapist should conclude that A. there is pressure dampening. B. the transducer is placed too high. C. there is an obstruction in the catheter. D. this is a normal wedge tracing.

Normal wedge tracing

Which of the following formulas will calculate the number of hours an E cylinder will provide oxygen to a patient? A. tank pressure (PSI) x 0.3 / flowrate (LPM) B. flowrate (LPM) x 0.3 / tank pressure (PSI) C. [tank pressure (PSI) x 0.3 / flowrate (LPM)] / 60 D. [flowrate (LPM) x 0.3 / tank pressure (PSI)] / 60

PSI x .3/LPM /60

A 34-week gestation age infant has just been delivered. The one and five minute apgar scores are 4 and 6. The physician has written an order for 40% humidified oxygen. Which of the following would be the most appropriate device? A. an incubator set at 40% oxygen B. a radient warmer set at 40% C. a oxygen hood set at 40% D. high humidity oxygen tent set at 40%

Oxygen hood set at 40%. stable environment

Which of the following will determine the partial pressure of oxygen in the alveolus? A. (Hb x 1.34 x SaO2) + (PaO2 x .003) B. (PaCO2 - PECO2) / PaCO2 C. (PAO2 - PaO2) .003 / [(PAO2 - PaO2) .003] + C(a-v)O2 D. (PB - PH2O) FIO2 - (PaCO2 / 0.8)

PAO2 equation

Which of the following will determine the oxygen gradient between the alveolus and the arterialized blood? A. PAO2 - PaO2 B. CaO2 - CvO2 C. (PaCO2 - PECO2) / PaCO2 D. CcO2 - CaO2 / CcO2 - CvO2

PAO2-PaO2

A 58-year-old woman who weighs 65 kg (143 lb) arrives in the surgical ICU following surgical repair of an abdominal aortic neurysm. She has a 60 pack year history of cigarette smoking and is currently smoking 1 pack/day. The physician has written orders to initiate mechanical ventilation. Prior to starting mechanical ventilation for this patient, the respiratory therapist should set all of the following controls EXCEPT A. FIO2. B. tidal volume C. frequency. D. PEEP.

PEEP

A 58-year-old woman who weighs 65 kg (143 lb) arrives in the surgical ICU following surgical repair of an abdominal aortic neurysm. She has a 60 pack year history of cigarette smoking and is currently smoking 1 pack/day. The physician has written orders to initiate mechanical ventilation. Prior to starting mechanical ventilation for this patient, the respiratory therapist should set all of the following controls EXCEPT A. FIO2. B. tidal volume C. frequency. D. PEEP

PEEP

Which of the following is NOT a primary ventilation parameter measured and displayed to create airway graphics of continuous mechanical ventilation? A. Flow B. PEEP C. Tidal volume D. Airway pressure

PEEP

Which of the following is NOT a primary ventilation parameter measured and displayed to create airway graphics of continuous mechanical ventilation? A. Flow B. PEEP C. Tidal volume D. Airway pressure

PEEP

Following exploratory laparotomy, a 51-year-old man has retained secretions in his left lower lobe. The respiratory therapist should recommend A. blow bottles. B. NIPPV (BiPAP®). C. albuterol MDI with spacer. D. PEP therapy.

PEP therapy

While assessing a patient's breath sounds the respiratory therapist notes that when the patient is instructed to say the letter "E", it comes through the stethescope sounding like "aaaahhh". This change in the sound is associated with which of the following conditions? A. Pleuritic inflammation B. Pneumonia C. Bronchospasm D. Epiglotitis

PNA

While assessing a patient's breath sounds the respiratory therapist notes that when the patient says "ninety-nine", it sounds very loud through the stethescope. This would be associated with which of the following conditions? A. Pneumothorax B. Pneumonia C. Pleural effusion D. Epiglottitis

PNA

An 8-year-old patient with asthma has been receiving routine bronchodilator therapy at home. The degree of airflow response to the therapy is best assessed by which of the following? A. Wright respirometer B. SpO2 C. Vital capacity D. Peak flow meter

Peak Flow Meter

A chest radiograph is taken in the lateral decubitus position in order to identify which of the following pathologies? A. Pleural effusion B. Pulmonary embolus C. Pneumothorax D. Pneumonia

Pleural Effusion

A chest radiograph is taken in the lateral decubitus position in order to identify which of the following pathologies? A. Pleural effusion B. Pulmonary embolus C. Pneumothorax D. Pneumonia

Pleural Effusion

The results of a patient's chest radiograph reveal the presence of a left basilar free fluid accumulation with a meniscus formation. Physical examination of the chest indicates a dull percussion note on the left and trachea shifted to the right. These results are consistent with which of the following conditions? A. Pleural effusion on the left B. Basilar pneumonia of the left lung C.atelectasis of the left lung D. Pneumothorax in the left lung

Pleural effusion on the left

While assessing a patient's breath sounds the respiratory therapist notes that when the patient says "ninety-nine", it sounds very loud through the stethescope. This would be associated with which of the following conditions? A. Pneumothorax B. Pneumonia C. Pleural effusion D. Epiglottitis

Pneumonia.- loud 99 means there is consolidation in lungs

The best way to check the accuracy of an air/oxygen proportioner is by using: A. polarographic electrode analyzer B. precision geisler tube analyzer C. infrared absorption analyzer D. teflon membrane analyzer

Polarographic electrode analyzer measures O2. Geisler= Nitrogen. Infrared=CO2. Teflon=ABG analyzer

The following data has been obtained from a 28-week gestational age infant who was born premature: Color: Cyanotic Chest x-ray: Cardiac enlargement Chest Sounds: Systolic murmur Respiratory rate: 55 Br/min. SpO2: 80% Which of the following diagnostic tests should the respiratory therapist recommend? A. Pre- and post-ductal blood gas studies B. L/S ratio C. New Ballard Score D. Capillary blood gas

Pre and Post ductal blood gas

The respiratory therapist is assisting the physician with the insertion of a pulmonary artery catheter. The therapist and physician have proper sterile gloves and gown, and a sterile field is arranged. As the therapist hands the pulmonary artery catheter to the physician, the therapist drops the catheter onto the sterile field. The respiratory therapist should A. run to get another sterile catheter. B. proceed with the insertion of the catheter. C. rinse the catheter with sterile water. D. ly inspect the catheter for any debris.

Proceed with the insertion of the catheter

A patient is on a 28% Venturi mask with the bed covers pulled up over the mask. Arterial blood gas results on the 28% Venturi mask show the PaO2 is 168 torr. The physician asks the respiratory therapist for a recommendation. Which of the following should the therapist recommend? A. Pull the covers down away from the mask and repeat the arterial blood gas in 30 minutes B. Discontinue the oxygen therapy C. Switch the patient to a nasal cannula at 2 L/min D. Report the arterial blood gas results as

Pul the covers down away from the mask and repeat the ABG in 30 min

The tip of a catheter used to measure the wedge pressure should be positioned in the: A. superior vena cava B. right atrium C. pulmonary artery D. pulmonary vein

Pulmonary Artery

The tip of a catheter used to measure the wedge pressure should be positioned in the: A. superior vena cava B. right atrium C. pulmonary artery D. pulmonary vein

Pulmonary Artery

A pleural friction rub is associated with all of the following conditions EXCEPT A. pneumonia. B. tuberculosis. C. pleurisy. D. pulmonary edema.

Pulmonary Edema

A pleural friction rub is associated with all of the following conditions EXCEPT A. pneumonia. B. tuberculosis. C. pleurisy. D. pulmonary edema.

Pulmonary Edema. Has fluid that would not have a pleural friction

A patient in the intensive care unit has the following hemodynamic measurements: CVP (cm H2O): 12 PAP (mm Hg): 48/25 PCWP (mm Hg): 8 MAP (mm Hg): 93 Cardiac output (L/min): 6.1 Cardiac index (L/min/m2): 3.2 These results are consistent with: A. Hypervolemia B. Congestive heart failure C. Pulmonary hypertension D. Cor pulmonale

Pulmonary Hypertension

A patient in the intensive care unit has the following hemodynamic measurements: CVP (cm H2O): 12 PAP (mm Hg): 48/25 PCWP (mm Hg): 8 MAP (mm Hg): 93 Cardiac output (L/min): 6.1 Cardiac index (L/min/m2): 3.2 These results are consistent with: A. Hypervolemia B. Congestive heart failure C. Pulmonary hypertension D. Cor pulmonale

Pulmonary Hypertension

A 56-year-old male patient who weighs 90 kg (198 lb) is receiving mechanical ventilation in the PC, SIMV mode and the following data is available: FIO2: 1.00 Set rate: 20 /min. Total rate: 20 /min. PEEP: 18 cm H2O Peak pressure: 35 cm H2O pH: 7.35 PaCO2: 45 torr PaO2: 43 torr HCO3-: 22 mEq/L It would be appropriate for the respiratory therapist to A. increase the set rate to 25 /min. B. switch to inverse ratio ventilation. C. increase pressure to 40 cm H2O. D. increase PEEP to 25 cm H2O.

Switch to inverse ratio ventilation. Patient is shunting severe ARDS

Which of the following parameters are acceptable for weaning a patient from continuous mechanical ventilation? 1. VD/VT: 65% 2. MIP: -18 cm H2O 3. Qs/Qt: 10% 4. A-aDO2: 12 torr (21% oxygen) A. 3 & 4 only B. 1 & 2 only C. 1, 2 & 3 only D. 1, 2, 3 & 4

Qs/Qt: 10% (<20%-normal A-aDO2:12 torr (21% O2) Difference should always be less than FiO2

Which of the following parameters are acceptable for weaning a patient from continuous mechanical ventilation? 1. VD/VT: 65% 2. MIP: -18 cm H2O 3. Qs/Qt: 10% 4. A-aDO2: 12 torr (21% oxygen) A. 3 & 4 only B. 1 & 2 only C. 1, 2 & 3 only D. 1, 2, 3 & 4

Qs/Qt:10% & A-aDO2:12 torr (21%O2)

A patient is being ventilated with a Servo 300 ventilator in the intensive care unit. The following data is obtained: Mode: SIMV Mandatory rate: 12 b/min Total rate: 12 b/min VE: 8.6 L FIO2: 0.60 PIP: 31 cm H2O PEEP: 10 cm H2O pH: 7.41 PaCO2: 40 torr PaO2: 95 torr SaO2: 96% HCO3-: 23 mEq/L BE: +1 mEq/L A-aDO2: 300 torr C(a-v)O2: 3.6 vol% What should the therapist report as the QS/QT? A. 15% B. 18% C. 20% D. 25%

Qs/Qt= 20% normal shunt of 5% (+5% for each 100 shunt)

During a 6 Minute Walk Test, the patient should be instructed to A. rest for six minutes every half hour. B. record the distance he traveled in six minutes. C. count how many steps he took in six minutes. D. measure his heart rate every six minutes.

Record the distance he traveled in six min

A mask CPAP system using a threshold resistor valve is not maintaining the desired pressure. To correct this problem, the respiratory therapist should do all of the following EXCEPT: A. check the valve for proper function. B. reduce gas inlet flow. C. reposition the mask and check the seal. D. add a reservoir to the system.

Reduce gas inlet flow

A post-operative thoracotomy patient is receiving mechanical ventilation in the recovery room with a tidal volume of 750 mL, SIMV mandatory rate 8/min, FIO2 of 0.40. Arterial blood gas results show: pH: 7.36 PaO2: 89 torr PaCO2: 45 torr SaO2: 95% The patient is breathing fast and shallow. Based upon this information, the respiratory therapist should recommend A. repeating the arterial blood gas in 30 minutes. B. increase SIMV mandatory rate to 10/min. C. sedating the patient. D. paralyzing the patient with pancuronium bromide (Pavulon).

Repeat the ABG in 30 min.- Current ABG is fine, patient not ready to come off ventilator yet

A patient is receiving BiPAP® therapy and is complaining of eye irritation. What should the therapist do to correct the problem? A. Decrease the pressure B. Reassure the patient C. Reposition the mask D. Intubate the patient

Reposition the Mask

A patient is receiving BiPAP® therapy and is complaining of eye irritation. What should the therapist do to correct the problem? A. Decrease the pressure B. Reassure the patient C. Reposition the mask D. Intubate the patient

Reposition the mask

A 26-year-old patient with shortness of breath is admitted to the emergency room. The patient states that he was running in Central Park with a friend and could not catch his breath. Bedside assessment reveals the following data: Pulse: 134 Respirations: 35 Color: pale SpO2: 91% on room air Breath sounds: diminished on the right The respiratory therapist should: A. request a STAT chest x-ray. B. recommend a V/Q scan C. insert a large bore needle into the 2nd intercostal space on the right side in the midclavicular line. D. insert a chest tube into the 2nd intercostal space on the right side in the midclavicular line

STAT Xray

Which of the following ventilator controls should NOT be adjusted if the I:E ratio alarm is activated with each mandatory breath? A. rate B. sensitivity C. volume D. flow

Sensitivity

Which of the following ventilator controls should NOT be adjusted if the I:E ratio alarm is activated with each mandatory breath? A. rate B. sensitivity C. volume D.flow

Sensitivity does not affect the I:E ratio

The respiratory therapist is assessing a patient's vital signs and notes that the pulse feels weak and thready. This would most likely be associated with which of the following conditions? A. Hypervolemia B. Shock C. Increased cardiac output D. Increased systemic vascular resistance

Shock

The respiratory therapist is assessing a patient's vital signs and notes that the pulse feels weak and thready. This would most likely be associated with which of the following conditions? A. Hypervolemia B. Shock C. Increased cardiac output D. Increased systemic vascular resistance

Shock

A 75-year-old patient with COPD is receiving oxygen at home by nasal cannula with a bubble humidifier. How should the respiratory therapist instruct the patient to clean his humidifier? A. Place it on the top shelf of the dishwasher. B. Soak it in an acetic acid solution for 20 minutes and rinse with water. C. Rinse it with distilled water and allow to air dry. D. Soak it in an alkaline glutaraldehyde solution for 30 minutes and rinse with water.

Soak it in acetic acid solution for 20 minutes and rinse with water. Easy for patient to do at home (vinegar and water)

Which of the following drugs should the respiratory therapist recommend to lower a patient's blood pressure as well as decrease his ventricular afterload? A. Norepinephrine B. Propranolol C. Procainamide D. Sodium nitroprusside

Sodium nitroprusside

Which of the following drugs should the respiratory therapist recommend to lower a patient's blood pressure as well as decrease his ventricular afterload? A. Norepinephrine B. Propranolol C. Procainamide D. Sodium nitroprusside

Sodium nitroprusside.

While performing an exercise stress test on a patient, the respiratory therapist observes a 6 mm depression in the ST segment. Which of the following should be done? A. Continue the test but at a slower pace B. Continue the test but monitor the patient closely C. Stop the test, stabilize the patient, and notify the physician D. Stop the test, let the patient rest for 5 minutes, then resume the test at the same work level

Stop the test, stabilize, and notify patient

The respiratory therapist reviews the ventilator flow sheet for a post-op patient. The ventilator settings have not been changed during the past 24 hours. The peak and plateau pressures were recorded as follows: PIP- 28,35,50 Pplat- 23,30,44 This information would indicate that A. airway resistance is increasing. B. lung compliance is increasing. C. the patient can begin weaning. D. the PEEP should be increased.

The PEEP should be increased. PIP-Pplat not changing so Raw not changing. Cst is decreasing

Which of the following would be the best method to deliver oxygen therapy to a full-term newborn? A. nasal cannula at 2 LPM B. croup tent at 30-40% oxygen C. oxyhood with a blender to adjust the FIO2 D. isolette with an oxygen bleed-in with a heated nebulizer

oxyhood with a blender to adjust FiO2

Which of the following statements is/are true regarding an Esophageal Tracheal Combitube? 1. Used for pre-hospital airway management. 2. Placed in the trachea with a laryngoscope. 3. It should be replaced with an endotracheal tube as soon as possible. A. 3 only B. 2 and 3 only C. 1 and 3 only D. 1 and 2 only

Used for pre-hospital airway management/replace with an endotracheal tube

The ICU director wishes to implement a protocol to reduce the risk of ventilator-associated pneumonia (VAP) for patients requiring mechanical ventilation. The respiratory therapist should recommend that the protocol include A. placing the patient in the prone position. B. changing the ventilator circuit each day. C. utilizing a closed-suction catheter system. D. intubating the patient with a Carlens tube.

Using a closed-suction catheter system

Which of the following is the best method to minimize damage to the tracheal wall caused by an endotracheal tube cuff? A. Measuring the volume used to inflate the cuff B. Palpating the inflation of the pilot balloon C. Utilizing a cuff pressure manometer D. Using minimal occluding volume to seal the airway

Utilizing a cuff pressure manometer

The respiratory therapist reviews the ventilator flow sheet for a post-op patient. The ventilator settings have not changed during the past 24 hours. The peak and plateau pressures were recorded as follows: PIP:28,35,40 Pplat:23,30,44 This information would indicate that A. airway resistance is increasing. B. lung compliance is decreasing. C. the patient needs to be suctioned. D. the patient can begin weaning.

The lung compliance is decreasing

A patient is receiving oxygen at home via a nasal cannula at 2 L/min connected to an oxygen concentrator. Humidification is being provided with a disposable bubble humidifier. The respiratory therapist is in the home for the patient's monthly visit and notices that the bubble humidifier is functioning properly, but the water in the humidifier reservoir appears to have a greenish tint. The patient is complaining of recurring infections. Which of the following should the therapist suspect? A. The patient is refilling the humidifier with tap water. B. There is a leak in the system. C. The water level is too high. D. The molecular sieve beds are obstructed.

The patient is refilling humidifier with tap water. Disposable -patient should throw away after each use

Oxygen concentrator operation involves all of the following EXCEPT A. removing nitrogen from the room air. B. electrically powered with limited portability. C. providing 100% oxygen at high flow rates. D.filters which require routine care.

They do not provide 100% O2 at high flow rates

A pH of 7.38, PaCO2 of 46 torr, PaO2 of 41 torr are obtained on a patient who appears to be healthy. There is no tachycardia, tachypnea, or cyanosis. Which of the following is an appropriate conclusion to draw on the basis of this information? A. The blood gas sample has a gas bubble B. The blood gas sample was not mixed or warmed prior to analysis C. This is a venous blood sample D. This patient has chronic lung disease

This is a venous blood sample

After injecting a small amount of air into the balloon of a pulmonary artery catheter, the respiratory therapist sees a small amplitude change with the mean pressure reading 2 points below the PA end-diastolic pressure. Based upon this information, the therapist should conclude that A. there is pressure dampening. B. the transducer is placed too high. C. there is an obstruction in the catheter. D. this is a normal wedge tracing.

Ths is a normal wedge tracing

While assisting a pulmonologist performing thoracentesis on a 58-year-old man, the respiratory therapist observes that the pleural fluid is clear with a light straw color. This fluid would best be described as A. exudate. B. transudate. C. purulent. D. serosanguineous.

Transudate

The following values were measured on a patient in the pulmonary function lab: FVC: 3.3, 3.4, 3.5 FEV1: 2.3, 2.5, 2.3 Which one represents the "best test"? A. Trial 1 B. Trial 2 C. Trial 3 D. The results of Trial 1 do not meet ATS standards and should be repeated

Trial 2. The number from FVC and FEV1 that add up to highest result

A tympanic percussion note is usually present in which of the following conditions? 1. gastric distension 2. pleural effusion 3. pneumothorax 4. endocarditis A. 1 & 3 only B. 2 & 4 only C. 3 & 4 only D. 1, 2 & 3 only

Typmpanic = air. Gastric and PNX

A 26-year-old patient with shortness of breath is admitted to the emergency room. The patient states that he was running in Central Park with a friend and could not catch his breath. Bedside assessment reveals the following data: Pulse: 120 Respirations: 25 br/min Color: pale SpO2: 89% on room air Breath sounds: slightly diminished on the right The respiratory therapist should: A. request a STAT chest x-ray. B. administer 100% oxygen. C. insert a large bore needle into the 2nd intercostal space on the right side in the midclavicular line. D. insert a chest tube into the 2nd intercostal space on the right side in the midclavicular line.

administer 100% O2.. Then get a XRAY if no improvement

A 28-week gestational age neonate is experiencing frequent apneic episodes with bradycardia. Which of the following should the respiratory therapist recommend? A. An FIO2 of 0.40 via oxyhood B. Suctioning the neonate during apnea episode C. Administering beclomethasone D. Administering theophylline

administer theophylline

A 2-year-old child has been admitted to the pediatric unit with a barking cough and mild stridor at rest. The pulse oximeter displays an SpO2 of 87% on room air. The physician asks the respiratory therapist to recommend the most appropriate oxygen delivery device for this child. The therapist should recommend a/an A. aerosol mask. B. nasal cannula. C. oxygen hood. D. venturi-mask.

aerosol mask. Signs of Croup. Cool aerosol will reduce swelling and O2 will increase SpO2

All of the following could be causing the peak pressure alarm to sound on a volume-cycled ventilator EXCEPT: A. airway resistance has decreased B. the patient needs to be suctioned C. compliance has decreased D. an insufficent amount of time for exhalation

airway resistance has decreased

Routes of transmission of infections include 1. airborne droplet nuclei. 2. contact. 3. respiratory droplets. A. 2 only B. 1 and 3 only C. 2 and 3 only D. 1, 2, and 3

all the above

A physician asks the respiratory therapist to measure a patient's ability to cough and clear secretions from his airway. Which of the following would the therapist utilize to evaluate the patient? A. wright respirometer B. flutter valve C. aneroid manometer D. spacer

aneroid manometer

A patient with COPD has been admitted for possible pneumonia. The patient is producing moderate amounts of thick yellow sputum and breath sounds are decreased in the right middle lobe. Sputum culture indicates a staphylococcal infection. Which of the following therapies should the respiratory therapist recommend? A. chest physical therapy B. antibiotic therapy C. incentive spirometry D. IPPB

antibiotic therapy

The respiratory therapist notices a dampened waveform on a pulmonary arterial line. The therapist's first action should be to A. attempt to draw blood from the line. B. check the transducer dome for air bubbles. C. flush the catheter with heparin solution. D. check the position of the transducer.

check the transducer dome for air bubbles

A patient is receiving heliox therapy via a simple mask. The patient's work of breathing has not decreased and the physician asks the respiratory therapist for a recommendation. The therapist should recommend: A. increasing the flow to the mask. B. decreasing the flow to the mask. C. changing to a nasal cannula. D. changing to a non-rebreathing mask.

change to a non-rebreathing mask

The respiratory therapist notices a dampened waveform on a pulmonary arterial line. The therapist's first action should be to A. attempt to draw blood from the line. B. check the transducer dome for air bubbles. C. flush the catheter with heparin solution. D. check the position of the transducer.

check the transducer dome for air bubbles

A 24-year-old woman with multiple trauma from a motor vehicle accident requires mechanical ventilation in the ICU. Because of a tension pneumothorax, a chest tube was inserted in the right second intercostal space and attached to a three-bottle water-seal drainage system with suction. While performing an assessment of the patient-ventilator system, the respiratory therapist notes continuous bubbling in the water seal bottle. The therapist should A. increase the suction pressure. B. clamp the chest tube. C. reduce the tidal volume. D. add water to the suction control bottle.

clamp the chest tube

A respiratory therapist is in the cafeteria when an adult visitor begins to choke. The therapist has administered 7 subdiaphragmatic thrusts without clearing the patient's airway, although the patient remains conscious. The therapist should A. administer 5 back blows. B. continue subdiaphragmatic thrusts. C. attempt to ventilate. D. check for presence of a pulse.

continue subdiaphragmatic thrusts

A respiratory therapist is in the cafeteria when an adult visitor begins to choke. The therapist has administered 7 subdiaphragmatic thrusts without clearing the patient's airway, although the patient remains conscious. The therapist should A.adinister 5 back blows. B. continue subdiaphragmatic thrusts. C. attempt to ventilate. D. check for presence of a pulse.

continue thrusts

A 24-year-old post-operative male patient is receiving intermittent positive pressure therapy at 20 cm H2O. The patient complains that the machine is cycling off too soon. The patient's post-operative spontaneous vital capacity is 3.5 L. Which of the following should the respiratory therapist recommend? A. Increase the pressure to 25 cm H2O B. Discontinue therapy, encourage deep breathing and coughing frequently on his own C. Switch to a volume incentive spirometry device D. Recommend decreasing the flow

discontinue the therapy, encourage deep breathing and coughing frequently

The advantages of an electronic medical record include all of the following EXCEPT A. allows the respiratory therapist to treat multiple patients simultaneously. B. improves the accuracy of charting the results of therapy. C. data entry is more efficient. D. prevents unauthorized access to medical information.

doesn't allow the respiratory therapist to treat multiple patients simultaneously

While performing routine ventilator parameter checks on a patient on a microprocessor ventilator with a wick humidification system, the respiratory therapist notices there is very little condensation in the tubing. The most likely explanation is that the A. temperature probe is placed distal to the wye ad adapter. B. room temperature is lower than normal. C. heating element is not functioning properly. D. water level is just slightly above the refill line.

heating element is not functioning properly

While performing routine ventilator parameter checks on a patient on a microprocessor ventilator with a wick humidification system, the respiratory therapist notices there is very little condensation in the tubing. The most likely explanation is that the A. temperature probe is placed distal to the wye adapter. B. room temperature is lower than normal. C. heating element is not functioning properly. D. water level is just slightly above the refill line.

heating element not functioning properly

A 57-year-old male patient is being mechanically ventilated following removal of his right upper lobe. The chest radiograph shows right side consolidation. An arterial blood gas has been obtained. Ventilatory data and blood gas results are: Mode: VC, SIMV FIO2: 1.0 Set rate: 20 br/min Total rate: 20 br/min VT: 450 mL PIP: 55 cm H2O PEEP: 18 cm H2O pH: 7.41 PaCO2: 39 torr PaO2: 43 torr HCO3: 23 mEq/L BE: 0 mEq/L The respiratory therapist should recommend A. independent lung ventilation. B. inverse ratio ventilation. C. pressure support ventilation. D. pressure control ventilation.

independent lung ventilation

A 32-year-old patient with shortness of breath is admitted to the emergency room. The patient states that she was running with a friend and suddenly felt pain on the right side of her chest. Bedside assessment reveals the following data: Heart rate: 148 Respirations: 44 br/min Color: cyanotic SpO2: 70% on room air Breath sounds: absent on the right Palpation: trachea deviated to the left Percussion: tympany on the right side The respiratory therapist should first: A. request a STAT chest x-ray. B. administer IPPB stat. C. insert a 14 gauge needle into the 2nd intercostal space on the right side in the midclavicular line. D. insert a chest tube into the 2nd intercostal space on the right side in the midclavicular line.

insert a 14 gauge needle into the 2nd intercostal space on the right side in midclavicular line

A 32-week gestational age infant is receiving mechanical ventilation for hyaline membrane disease. The respiratory therapist suspects that a pneumothorax has developed and performs transillumination, which reveals a brightly illuminated left thorax. The respiratory therapist's FIRST action should be to A. insert a chest tube and connect to a pleural suction system. B. perform a fiberoptic bronchoscopy. C. obtain a STAT chest film. D. suction the infant.

insert a chest tube and connect to a pleural suction system

While performing nasotracheal suctioning on a 64-year-old patient with pneumonia, the respiratory therapist notes that the patient's nares have developed redness and swelling. To prevent further irritation, the respiratory therapist should recommend? A. switching to orotracheal suctioning B. inserting an orotracheal tube C. administering an antibiotic to the site D. inserting a nasopharyngeal airway

insert a nasopharygngeal airway

While performing nasotracheal suctioning on a 64-year-old patient with pneumonia, the respiratory therapist notes that the patient's nares have developed redness and swelling. To prevent further irritation, the respiratory therapist should recommend? A. switching to orotracheal suctioning B. inserting an orotracheal tube C. administering an antibiotic to the site D. inserting a nasopharyngeal airway

inserting a nasopharyngeal airway

During an incentive spirometry treatment using a volume-oriented device, the patient inhales to TLC and exhales slowly to FRC. What other instructions would you give to improve distribution? A. continue with the current instructions B. switch to a flow orientated device C. initiate IPPB D. inspiratory hold

inspiratory hold. This will improve ventilation distribtution

A 52-year-old, 5' 9" tall male patient weighing 85 kg (187 lb) is being mechanically ventilated. An arterial blood gas has been obtained. Ventilatory data and blood gas results are below: Mode: VC, A/C FIO2: 0.40 Mandatory rate: 16 /min Total rate: 16 /min VT: 800 mL PEEP: 5 cm H2O pH: 7.47 PaCO2: 31 torr PaO2: 87 torr SaO2: 96% HCO3-: 23 mEq/L BE: 0 mEq/L Which of the following changes should be recommended at this time? A. Increase the rate to 20 /min B. incrasePEEP 10 cm H2O C. Add 200 mL of mechanical deadspace D. Maintain current settings

maintain the current settings

A patient with a size 8.5 mm oral endotracheal tube is transported from the ED to the ICU. The respiratory therapist suspects that the endotracheal tube has changed position during transport. Which of the following is the quickest way to assess the position of the tube? A. Chest radiograph B. Auscultation of the chest C. Observation of chest movement D. Diagnostic chest percussion

observation of chest movement

A patient with a size 8.5 mm oral endotracheal tube is transported from the ED to the ICU. The respiratory therapist suspects that the endotracheal tube has changed position during transport. Which of the following is the quickest way to assess the position of the tube? A. Chest radiograph B. Auscultation of the chest C. Observation of chest movement D. Diagnostic chest percussion

observe chest movement

While performing oxygen rounds, the respiratory therapist enters a patient's room and hears a high pressure pop-off alarm sounding from a bubble humidifier. Which of the following is the most likely explanation for this? A. Obstruction in the connecting line B. Leak in the humidifier C. Problem with the valve seat in the flowmeter D. Flowmeter is uncompensated for back pressure

obstruction in the connecting line. occlude the connecting tube to check for pop off alarm. or O2 could be high

A patient with a history of COPD is receiving 28% oxygen therapy via air-entrainment mask. While performing oxygen rounds, the respiratory therapist notes that the device is delivering 40% oxygen. Which of the following conditions is the most likely cause? A. increase in oxygen flow to the dilution jet B. increase in size of air entrainment ports C. obstruction in the face mask D. decrease in size of jet orifice

obstruction in the face mask. smaller orifice=more air entrained, larger orifice=less air entrained=higher fio2. larger port=more air < fio2. smaller port= less air >fio2

Therapy for sleep study Nasal airflow-decreased or no airflow Thoracic chest movement-continuous

obstructive sleep apnea when we have continuous movement of chest w/no airflow. Initial TX is nasal Cpap

A 34-week gestation age infant has just been delivered. The one and five minute apgar scores are 4 and 6. The physician has written an order for 40% humidified oxygen. Which of the following would be the most appropriate device? A. an incubator set at 40% oxygen B. a radient warmer set at 40% C. a oxygen hood set at 40% D. a high humidity oxygen tent set at 40%

oxygen hood set at 40%. Attached to a heated humidifer. place hood inside of incubator to keep neutral thermal environment

A patient is receiving oxygen at home via a nasal cannula at 2 L/min connected to an oxygen concentrator. Humidification is being provided with a disposable bubble humidifier. The respiratory therapist is in the home for the patient's monthly visit and notices that the bubble humidifier is functioning properly, but the water in the humidifier reservoir appears to have a greenish tint. The patient is complaining of recurring infections. Which of the following should the therapist suspect? A. The patient is refilling the humidifier with tap water. B. There is a leak in the system. C. The water level is too high. D. The molecular sieve beds are obstructed.

patient is refilling humidifier with tap water

An oxygen conservation cannula would be most appropriate for which of the following patients? A. A patient with pneumonia requiring low flow oxygen. B. A patient with emphysema who is experiencing increased shortness of breath. C. A patient with a severe exacerbation of asthma. D. A patient with pulmonary fibrosis requiring long term oxygen therapy.

patient with pulmonary fibrosis requiring long term O2 therapy

A 2 kg (4.4 lb) neonate requires transportation to a tertiary care center for cardiac surgery. The infant has a heart rate of 140 and a BP of 60/30.The neonate is intubated with a size 2.5 mm uncuffed endotracheal tube and ventilation is being assisted manually at a rate of 40/min. Which of the following should the respiratory therapist recommend? A. postpone the transport until the neonate has spontaneous respirations B. prepare for the transport immediately C. delay the transport and stabilize the patient D. wait 24 hours and then reassess the patient

prepare to transprt

Oxygen concentrator operation involves all of the following EXCEPT A. removing nitrogen from the room air. B. electrically powered with limited portability. C. providing 100% oxygen at high flow rates. D. filters which require routine care.

providing 100% O2 at high flow rates. Won't always get 100%

A post-operative thoracotomy patient is receiving mechanical ventilation in the recovery room with a tidal volume of 750 mL, SIMV mandatory rate 8/min, FIO2 of 0.40. Arterial blood gas results show: pH: 7.36 PaO2: 89 torr PaCO2: 45 torr SaO2: 95% The patient is breathing fast and shallow. Based upon this information, the respiratory therapist should recommend A. repeating the arterial blood gas in 30 minutes. B. increase SIMV mandatory rate to 10/min. C. sedating the patient. D. paralyzing the patient with pancuronium bromide (Pavulon).

repeat the ABG in 30 min. (current bood gasses don't warrant any change at this time)

A 58-year-old female patient who weighs 165 lb (75 kg) is orally intubated with a size 6.0 mm endotracheal tube and is receiving volume-control ventilation. The tube is taped at the 23 cm mark at the patient's lips. The tube cuff pressure measures 38 mm Hg. When the respiratory therapist reduces the cuff pressure, the low exhaled volume alarm on the ventilator begins to sound. The therapist should A. replace the endotracheal tube with a larger size. B. maintain the cuff pressure at 38 mm Hg. C. deflate the cuff and advance the tube. D. adjust the low exhaled volume alarm.

replace the ETT with a larger size. 75 kg(decimal in middle) 7.5 ETT

A 58-year-old female patient who weighs 165 lb (75 kg) is orally intubated with a size 6.0 mm endotracheal tube and is receiving volume-control ventilation. The tube is taped at the 23 cm mark at the patient's lips. The tube cuff pressure measures 38 mm Hg. When the respiratory therapist reduces the cuff pressure, the low exhaled volume alarm on the ventilator begins to sound. The therapist should A. replace the endotracheal tube with a larger size. B. maintain the cuff pressure at 38 mm Hg. C. deflate the cuff and advance the tube. D. adjust the low exhaled volume alarm.

replace the endotracheal tube with a larger size

A 26-year-old patient with shortness of breath is admitted to the emergency room. The patient states that he was running in Central Park with a friend and could not catch his breath. Bedside assessment reveals the following data: Pulse: 134 Respirations: 35 Color: pale SpO2: 91% on room air Breath sounds: diminished on the right The respiratory therapist should: A. request a STAT chest x-ray. B. recommend a V/Q scan C. insert a large bore needle into the 2nd intercostal space on the right side in the midclavicular line. D. insert a chest tube into the 2nd intercostal space on the right side in the midclavicular line

request stat xray. we need more info

A 44-year-old woman has just undergone a cholecystectomy. Over the last 48 hours the patient has the following laboratory findings: K+: 3 mEq/L Na+: 115 mEq/L Cl-: 80 mEq/L HCO3-: 24 mEq/L Urine output: 60 mL/hour BP: 125/95 mm Hg Based upon this information the respiratory therapist would conclude A. the patient is hyperkalemic B. the patient requires decreased fluid intake C. the patient is polycythemic D. the patient has a metabolic alkalosis

requires decreased fluid intake

The results of a patient's chest radiograph reveal the presence of a right side consolidation with a concave-shape appearance and a mediastinal shift to the left. Physical examination of the chest reveals decreased movement, flat percussion, and absent breath sounds over the right lower chest. These results are consistent with which of the following conditions? A. right side pleural effusion B. basilar pneumonia of the right lung C. atelectasis of the right lower lobe D. right side tension pneumothorax

right side pleural effusion

The results of a patient's chest radiograph reveal the presence of a right side consolidation with a concave-shape appearance and a mediastinal shift to the left. Physical examination of the chest reveals decreased movement, flat percussion, and absent breath sounds over the right lower chest. These results are consistent with which of the following conditions? A. right side pleural effusion B. basilar pneumonia of the right lung C. atelectasis of the right lower lobe D. right side tension pneumothorax

right side pleural effusion.

A male patient (6' 2" tall, 80 kg, 176 lb) is being mechanically ventilated following an exploratory laporatomy at the following settings: Mode: VC, A/C Set rate: 12 /min. Total rate: 18 /min. FIO2: 0.50 VT: 700 mL PEEP: 6 cm H2O pH: 7.49 PaCO2: 32 torr PaO2: 90 torr HCO3-: 24 mEq/L Based on his information, the respiratory therapist should A. add mechanical deadspace. B. change to VC, SIMV. C. decrease the FIO2 to 0.45. D. initiate pressure support.

switch to VC, SIMV- patient has 6 breaths over set rate that are full positive pressure breath causing hyperventilation

A 14-year-old patient has a size 7 mm endotracheal tube placed orally. While suctioning this patient with a size 12 French catheter you meet resistance while attempting to pass the catheter through the tube. The respiratory therapist should A extubate and reintubate with a size 8 mm endotracheal tube. B. switch to a size 10 French catheter. C. instill mucomyst down the endotracheal tube. D. lubricate the catheter with K-Y jelly prior to suctioning.

switch to a size 10 French catheter

A 14-year-old patient has a size 7 mm endotracheal tube placed orally. While suctioning this patient with a size 12 French catheter you meet resistance while attempting to pass the catheter through the tube. The respiratory therapist should A. extubate and reintubate with a size 8 mm endotracheal tube. B. switch to a size 10 French catheter. C. instill mucomyst down the endotracheal tube. D. lubricate the catheter with K-Y jelly prior to suctioning.

switch to a size 10 french catheter

A patient with a history of myasthenia gravis has just been admitted for increased muscle weakness. The respiratory therapist should recommend which of the following diagnostic tests to monitor the patient's drug therapy? A. polysomnography B. electroencephalography C. tensilon challenge test D. methacholine challenge test

tensilon challenge test

What would the therapist palpate when assessing a patient with a tension pneumothorax? A. hyperresonance B. tracheal deviation C. absent breath sounds D. increased work of breathing

tracheal deviation

What would the therapist palpate when assessing a patient with a tension pneumothorax? A. hyperresonance B. tracheal deviation C. absent breath sounds D. increased work of breathing

tracheal deviation

Which of the following is the best method to minimize damage to the tracheal wall caused by an endotracheal tube cuff? A. Measuring the volume used to inflate the cuff B. Palpating the inflation of the pilot balloon C. Utilizing a cuff pressure manometer D. Using minimal occluding volume to seal the airway

use a cuff pressure manometer

A respiratory therapist working in the intensive care unit would use a stylet to: A. assist with phonation B. facilitate orotracheal intubation C. maintain a stoma opening D. provide protection to the unaffected lung

used for intubation


Conjuntos de estudio relacionados

Lesson 5: Summarizing Basic Cryptographic Concepts

View Set

Post-class Multiple Choice Chp. 4

View Set

IP Subnetting, Troubleshooting IP- Chapter 8

View Set

TeXes Social Studies 4-8 (118) Prep

View Set